UWorld first half Flashcards

1
Q

Dynein vs Kinesin

A
  • Dynein is RETROGRADE transport down an axon (would be used to establish latent infection of HSV)
  • Kinesin is ANTEROGRADE transport down an axon (would be used to RE-activate HSV from cell nucleus down to skin, if HSV-1, would be trigeminal, if HSV2 would be from sacral ganglion, if VZV would be from DRG)
How well did you know this?
1
Not at all
2
3
4
5
Perfectly
2
Q

These are receptors for what viruses?

  1. Cellular Integrin
  2. CR2 (CD21)
  3. CD4 and CXCR4/CCR5
  4. Nicotinic ACh Receptor
  5. ICAM 1 (CD54)
A
  1. Cellular Integrin –> Cytomegalovirus
  2. CR2 (CD21) –> EBV
  3. CD4 and CXCR4/CCR5 –> HIV
  4. Nicotinic ACh Receptor –> Rabies
  5. ICAM 1 (CD54) –> Rhinovirus (think photos of rhino)
How well did you know this?
1
Not at all
2
3
4
5
Perfectly
3
Q

Scapular Winging

A

Lesion of LONG THORACIC nerve usually seen after masectomy, axillary node dissection and stab wounds.

-Defect in SERRATUS ANTERIOR (abducts arm above horizontal plane)

How well did you know this?
1
Not at all
2
3
4
5
Perfectly
4
Q

Calculating Number Needed to Harm (NNH)

A

1/Attribuatable Risk

Calculate attributable risk: take the adverse pts/total pts for both treatment and placebo. Then take the difference of these two numbers and take 1/AR

Ex: 60/80=.75 (75%) and 38/76 =.5 (50%) then take the difference of these two .75-.5=.25. Then take 1 over this number 1/.25 =4

How well did you know this?
1
Not at all
2
3
4
5
Perfectly
5
Q

Trigeminal Neuralgia (tic douloureux)

A

Sudden severe pain the distribution of CN V that can be brought on by chewing or brushing teeth. Unknown etiology

Tx: Carbamezapine: inhibits neuronal high frequency firing (a drug used for simple, complex, and tonic clonic seizures)

Baclofen and Valproic Acid can be used but are not first line

How well did you know this?
1
Not at all
2
3
4
5
Perfectly
6
Q

JVD wave

A

A. Right atrial contraction

C. bulging of R tricuspid valve during ventrical contraction

X: R atrial relaxation

V: continued inflow of venous blood

Y: passive emptying of R atrium after tricuspid opens

How well did you know this?
1
Not at all
2
3
4
5
Perfectly
7
Q

DNA Polymerase I vs III

A

DNA Polymerase I: Degrades RNA primer, replaces it with DNA

DNA Polymerase III: elongates DNA strand by adding deoxynucleotides to 3’ end. Elongates lagging strand and proofreads. Replicates the E. Coli chromosome

“333 for eeeecoli”

How well did you know this?
1
Not at all
2
3
4
5
Perfectly
8
Q

Immediate vs Long-term treatment for Panic Disorder

A

Acute: Benzodiazapines are OK for acute relief but should not be used long term due to risk of abuse. Contraindicated if have a hx of any sort of substance abuse

Chronic: SSRI/SNRI/CBT because of the relative benign side effect profile

How well did you know this?
1
Not at all
2
3
4
5
Perfectly
9
Q

Primase

A

-Makes an RNA primer on whiche DNA Polymerase III can initiate replication

*Keep in mind Uracil is ONLY found on RNA so if it were present, know we must be talking a bout RNA and thus primase would be involved

-Primase is crucial for bacterial replication as DNA polymerase CANNOT initiate DNA synthesis without it

How well did you know this?
1
Not at all
2
3
4
5
Perfectly
10
Q

Staphalococcal Scalded Skin Syndrome

A

-Caused by exotoxin mediated skin damage (exofoliatin exotoxin)

Presents with + Nokolsky sign, epidermal necrolysis, fever, pain

-Most common in infants and young children

How well did you know this?
1
Not at all
2
3
4
5
Perfectly
11
Q

S1 nerve root impingement

A
  • sensory loss of buttox, posterior thigh and calf, and lateral foot
  • weakness in hip extension (glut maximus), ACHILLES reflex, foot plantarflexion (gastroc), knee flexion (hamstrings)
How well did you know this?
1
Not at all
2
3
4
5
Perfectly
12
Q

L4 nerve root compression

A
  • sensory loss over lower anteriomedial thigh
  • weakness hip adduction, knee extension (quads), PATELLAR reflex
How well did you know this?
1
Not at all
2
3
4
5
Perfectly
13
Q

Loss of patellar reflex is compression at ___, loss of achilles reflex is compression at ___

A

Patellar –> L4

Achilles –> S1

“one/two, tie my shoe, three/four kick the door, five/six pick up sticks seven/eight shut the gate

How well did you know this?
1
Not at all
2
3
4
5
Perfectly
14
Q

Bacterial pneumonia AFTER infleunza infection is typically what bacteria

A

S. Aureus

How well did you know this?
1
Not at all
2
3
4
5
Perfectly
15
Q

CAAT box

A

highly conseved sequence that serves as a promoter of transcription in the eukaryotic genome

How well did you know this?
1
Not at all
2
3
4
5
Perfectly
16
Q

Chlorthalidone

A
  • potent thiazide diuretic (inhibit Na/Cl cotransporter in DCT)
  • results in hypokalemia and metabolic acidosis (via activation of the RAAS system and losing of K)

Hypokalemia can cause muscle weakness, cramps, rhabdomyolysis

How well did you know this?
1
Not at all
2
3
4
5
Perfectly
17
Q

involuntary head bobbing

A

-usually caused by aortic regurgitation which results in a widened pulse pressure (peak systolic - end diastolic)

How well did you know this?
1
Not at all
2
3
4
5
Perfectly
18
Q

DIC in pregnancy

A
  • mediated by release of tissue factor (thromboplastin) from sites of placental injury, which initiates the clotting cascade. Clotting factors are used up and this results in profound bleeding
  • An abruption that leads to fetal demise is the most common cause of DIC in pregancy
How well did you know this?
1
Not at all
2
3
4
5
Perfectly
19
Q

pt presents with acanthosis nigricans what is the most likely diagnosis and what is elevated in serum

A
  • Type II DM
  • Free fatty acids are elevated in serum

insulin resistance in adipose cells hinders antilipolytic effects of insulin and therefore lipolysis takes place and an increase in free fatty acid is observed. This contributes to the insulin resistance by impairing insulin-dependent glucose uptake and increasing hepatic gluconeogenesis

How well did you know this?
1
Not at all
2
3
4
5
Perfectly
20
Q

overdose of Beta-blocker medications

A
  • Presents as hypotensive and bradycardic on physcial exam (due to non-selective blockage of peripheral beta adrenergic receptors causing depression of myocardial contractility, bradycardia, and varying degrees of AV block)
  • Tx with glucagon (increases cAMPm which increases intracellular Ca release leading to an increase in heart rate and cardiac contractility)
How well did you know this?
1
Not at all
2
3
4
5
Perfectly
21
Q

Colchicine mechanism

A
  • primarily it works by inhibiting microtubular polymerization (cannot form microtubules)
  • Also reduces the formation of Leukotriene B4

Adverse effects: nausea, diarrhea, abd pain

How well did you know this?
1
Not at all
2
3
4
5
Perfectly
22
Q

Atropine OD signs and symptoms and treatment

A
  • restlessness, disoriented, combative
  • peripherally can cause dry skin, hyperthermia, mydriasis, constipation (cant see, cant see, cant spit, cant shit)

Tx overdose with phyostigmine (not neostigmine because does not cross BBB into CNS due to quaternary structure)

How well did you know this?
1
Not at all
2
3
4
5
Perfectly
23
Q

Interscalene nerve block

A
  • used for procedures involving the neck and upper arm. Good way to access the brachial plexus roots and trunks
  • Causes transient IPSILATERAL DIAPHRAGMATIC PARALYSIS by anesthesizing the phrenic nerve root C3-C5
How well did you know this?
1
Not at all
2
3
4
5
Perfectly
24
Q

Rare but serious side effect of Trazodone

A
  • Priaprism
  • can also cause sedation, nausea, postural hypotension (remember blocks 5HT2 and A1 adrenergic
How well did you know this?
1
Not at all
2
3
4
5
Perfectly
25
Q

Temporary local hyperoxia (like seen in a premature infant given high doses of O2) results in what?

A

-Can induce VEGF in the retina resulting in neovascularizaion

=retinopathy of prematurity

How well did you know this?
1
Not at all
2
3
4
5
Perfectly
26
Q

What would be the use in these measurements?

  1. aPTT
  2. PT
  3. Bleeding time
  4. Fibrinogen Levels
  5. Fibrin Split Products
A
  1. aPTT: unfractionated heparin
  2. PT: Warfarin (WEPT)
  3. Bleeding Time: assessment of platelet function, would be increased in vWd, ASA therapy, DIC,
  4. Fibrinogen Levels: would be low in DIC

5 Fibrin Split Products, would be low in DIC

How well did you know this?
1
Not at all
2
3
4
5
Perfectly
27
Q

What is so special about the longevity of EBV

A
  • when it infects B-cells via CD21, it stimulates them to enter the cell cycle and proliferate continuously (IMMORTAL). This happens by an EBV encoded oncogene activating proliferative and anti-apoptotic signaling pathways within the infected B-cell.
  • Keep in mind the dx of EBV is the production of heterophile antibodies (meaning react with foreign antigen such as hoarse erythrocytes
How well did you know this?
1
Not at all
2
3
4
5
Perfectly
28
Q

PFTs in Obstructive vs Restrictive Lung Disease

A

Restrictive Lung Disease: Decreased FEV1 but MORE of a decrease FVC leading to an INCREASE IN THE RATIO. Decreased TLC

“Restrictive Raise”

Obstructive: Decreased FVC but decreased MORE the FEV1 leading to a DECREASE IN THE RATIO. Increased TLC as well.

“Obstructive is Obsolete”

How well did you know this?
1
Not at all
2
3
4
5
Perfectly
29
Q

Electrolyte Disturbances of a CF pt?

A

-HYPOnatremic

  • They are salt wasting from sweat because sodium is NOT reabsorbed through the CFTR due to a mutation (deltaF508)
  • Risk factors for a CF pt would be excessive temperature, excessive exercise and the newborn period when they are solely on breastmilk and not eating salty foods yet (all pre-dispose to hyponatremia)
  • NOTE, these pts are also HYPOCHLOREMIC, but this is asymptomatic.
How well did you know this?
1
Not at all
2
3
4
5
Perfectly
30
Q

Where in the TCA cycle is GTP produced and what is it a subsequent cofactor for?

A
  • Succinyl CoA –> Succinate (near the bottom of the circle)
  • This is called substrate level phosphorylation and is carried out by the enzyme Succinyl CoA synthase
  • The GTP is then a cofactor for PEP-CK in gluconeogenesis going from oxaloacetate to PEP.
How well did you know this?
1
Not at all
2
3
4
5
Perfectly
31
Q

Absence of CD18

A

-Leukocyte Adhesion Deficiency (LAD). Defect in LFA1 integrin on CD18, leading to impaired migration and chemotaxis. CANNOT form integrins, which are essential for leukocyte adhesion

-presents with recurring skin and mucosal infections, and periodontal disease

Also see late separation of umbilical cord (>21 days)

How well did you know this?
1
Not at all
2
3
4
5
Perfectly
32
Q

What is the cause of the greenish color in bruises?

A
  • Erythrocytes escape into interstitum after injury and give blue/purple color
  • Erythrocyte destruction then causes release of heme molecules

HEME OXYGENASE comes along (inside macrophages) and converts Heme–>Billiverdin (which is green in color)

The Billiverdin is then further broken down to billirubin (yellow) and is then bound to albumin and taken to liver

How well did you know this?
1
Not at all
2
3
4
5
Perfectly
33
Q

Mechanism of abx resistance

  1. Penicillins
  2. Vancomycin
  3. Quinolones
  4. Aminoglycosides
  5. Tetracyclines
  6. Rifamycin
A
  1. PCN –> beta-lactamase, mutated PBP, mutated porin protein
  2. Vancomycin –> mutated peptidoglycan cell wall (d-ala, d-lac)
  3. Quinolones –> Mutated DNA gyrase, impaired influx/efflux
  4. Aminoglycosides –> bacterial transferase enzymes that acetylate, adenylate, or phosphorylate
  5. Tetracyclines –> influx/efflux
  6. Rifamycin –> mutated RNA polymerase
How well did you know this?
1
Not at all
2
3
4
5
Perfectly
34
Q

Spinal Muscular Atrophy=Werdnig Hoffmann Disease

A

Congenital degeneraton of anterior horns of spinal cord. LMN lesions resulting in “floppy baby” with marked hypotonia.

  • Caused by mutations in SMN1 gene which involves assembly of small nuclear ribonucleoproteins snRNP.
  • impaired SPLICESOME fxn (removes introns from pre-mRNA)
How well did you know this?
1
Not at all
2
3
4
5
Perfectly
35
Q

Uses of these monoclonal antibodies

  1. Infliximab
  2. Rituximab
  3. Interleukin 2
  4. Imatinib
  5. Abciximab
A

1. Infliximab: IgG1 monoclonal antibody against TNFalpha. Use in RA, AS, Crohns

2. Rituximab: targets CD20 B-cells used in lymphoma

3. Interleukin 2: cytokine that regulates activation and differentiation of T-cells. Use in Renal Cell Carcinoma and Melanoma

4. Imatinib: tyrosine kinase inhibitor used in CML

5. Abciximab: against platelet GP11B/IIIa receptor, used during angioplasty for pts with ACS

How well did you know this?
1
Not at all
2
3
4
5
Perfectly
36
Q

Finding of green, inspissated (thickened/congealed) mass in distal ilius in newborn points towards what and what disease associated with and what will this pt ultimately die from?

A

Inspissated mass (dehydrated meconoium) in distal ilium points to a diagnosis of meconium ileus which is very common in CF pts.

  • If you habe meconium ileus you will almost certainly be diagnosed with CF but if you have CF only about 10-20% of infants will have meconoim ileus
  • Ultimately CF pts usually die of pneumonia (or cor pulmonale, bronchiectasis)
How well did you know this?
1
Not at all
2
3
4
5
Perfectly
37
Q

What is NOT supplied in breastmilk and needs to be supplmenented by mother?

A

Vitamin K and Vitamin D

-The Vitamin K is supplemented by a shot at birth

-Vitamin D is usually supplemented by some sunlight (harder to get in AA babies because of an increase in melanocytes acting as a natural sunblock)

“KD for the KiD”

How well did you know this?
1
Not at all
2
3
4
5
Perfectly
38
Q

Pt with Rheumatic Fever is most likely to die from what?

A
  • Die from pancarditis (inflammation of the endocardium, myocardium, and pericardium)
  • Inflammation of the mitral valve is also common and will result in a holosystolic murmur
  • Severe regurgitation and myocarditis can lead to cardiac dilation, cardiac failure and DEATH in a very small number of pts
  • Pt will NOT die of septic shock from the S. aureus because the disease itself of rheumatic fever with migratory arthritis is autoimmune and NOT caused by the bacteria at this point.
How well did you know this?
1
Not at all
2
3
4
5
Perfectly
39
Q

Supprative Parotitis

A

-an acute viral or bacterial infection of the parotid gland

-S. Aureus is the most common cause.

-Risk factors include dehydration, intubation, anticholinergics, salivary calculi, or recent teeth cleaning

Dx: AMYLASE, US/CT

-Think about this in a pt who has acute jaw pain post intubation in a hospital

How well did you know this?
1
Not at all
2
3
4
5
Perfectly
40
Q

Treatment of BV

A

-Metronidazole or Clindamycin

Dx: grey or clear discharge with fishy smell. Use KOH to increase the smell “Whiff Test”

-Also dx with presence of CLUE CELLS

How well did you know this?
1
Not at all
2
3
4
5
Perfectly
41
Q

Diagnosis of Down Syndrome in vitro is suspected by?

A
  1. Decreased maternal serum AFP
  2. Increased nuchal translucency
  3. Increased b-hCG

It is then confirmed by chorionic villus sampling or amniocentesis

How well did you know this?
1
Not at all
2
3
4
5
Perfectly
42
Q

Why do you get oxalate kidney stones with Crohn’s Disease?

A

when the wall of the terminal ilium is inflamed, bile acids are not reabsorbed. This loss of bile acid reabsorption causes an decrease in fat absorption. These excess lipids in the bowel bind to calcium essentially performing a supponification reaction and these soap complexes are lost in the feces

-As a result, oxalate cannot be bound by calcium (like it noramally is) and is taken up in the bowel. This leads to an increase risk of kideny stones

How well did you know this?
1
Not at all
2
3
4
5
Perfectly
43
Q

Desflurane

A
  • A newer agent of the halothane class that is used for anesthetics and can lead to acute hepatitis.
  • It is associated with a centrilobular hepatic necrosis
  • Presents 2-3 days after exposure with fever, nausea, jaundice, scleral icterus, and increased liver enzymes
How well did you know this?
1
Not at all
2
3
4
5
Perfectly
44
Q

Ischemic Stroke Timeline

12-24 hours

24-72 hours

3-7 days

1-2 weeks

>2 weeks

A

12-24 hours –> “Red Neurons” (eosinophilic cytoplasm, loss of nissel substance)

24-72 hours –> neutrophilic infiltration

3-7 days –> macrophage/microglia infiltration and phagocytosis begin

1-2 weeks –> reactive gliosis and vascular proliferation around the necrotic area (liquefactive necrosis)

> 2 weeks –> glial scar formation

How well did you know this?
1
Not at all
2
3
4
5
Perfectly
45
Q

Pathology of a Hashimoto Thyroiditis pt

A
  • intense mononuclear infiltrate consisting of lymphocytes and plasma cells often with germinal centers.
  • Hurthle cells surround residual follicles
How well did you know this?
1
Not at all
2
3
4
5
Perfectly
46
Q

22q11.2 microdeletion

A

-DiGeorge Syndrome, neural crest cells fail to migrate into the 3rd and 4th pharyngeal pouches. Consequently end up with parathyroid and thymus hypoplasia. This results in hypocalcemia and T cell deficiency

-The hypocalcemia manifests as Chovstek and Troussou sign

How well did you know this?
1
Not at all
2
3
4
5
Perfectly
47
Q

Anencephaly

A

Failure of the anterior neuropore to close (can be prevented with folic acid supplementation during pregnacy)

How well did you know this?
1
Not at all
2
3
4
5
Perfectly
48
Q

Remnants of the Rathke Pouch could develop which tumor

A

-Craniopharyngioma resulting in hypopituitarism, hydrocephalus and diabetes insipitus

How well did you know this?
1
Not at all
2
3
4
5
Perfectly
49
Q

Post MI pt put on med with side effect of increased creatine kinase and muscle aches? what is med and what are some other side effects?

A
  • Pt is clearly on a statin, an HMG coA Reductase inhibitor and is having stain-induced myopathy
  • Statins block conversion from HMG CoA –> Mevalonic Acid, this decrease liver cholesterol leads to an increase in LDL clearance by the liver by LDL receptors. These LDL receptors endocytose the LDL and then are recycled for use again

also causes an upregaultion of HMG CoA reductase enzyme since it is all being blocked!

How well did you know this?
1
Not at all
2
3
4
5
Perfectly
50
Q

Latissimus Dorsi innervation and actions

A

Innervated by Thoracodorsal Nerve (from C6-C8 nerve roots)

Actions: Internal rotation of arm, extension, adduction of arm

A very vulnerable muscle to external injury

How well did you know this?
1
Not at all
2
3
4
5
Perfectly
51
Q

Refractory asthma treatment (when oral glucocorticoids and long acting beta adrenergic antagonist are NOT working)

A

-Omalizumab is an IgG monoclonal antibody that binds with IgE to inhibit the action of IgE on mast cells, basophils, or other cell types and decreases allergic response. (anti IgE antibody)

How well did you know this?
1
Not at all
2
3
4
5
Perfectly
52
Q

Canagliflozin

A

SGLUT-2 Inhibitor used for TII DM

Usually SGLUT-2 reabsorbs 90% of glucose from filtrate in the proximal tubule.

This drug leads to significant loss of urine glucose

Side effects: UTI’s (due to lots of glucose in urine feeding bacteria), systemic hypotension. Should be avoided in pts with moderate to severe renal impairment

-MUST CHECK RENAL FUNCTION PRIOR TO USE

How well did you know this?
1
Not at all
2
3
4
5
Perfectly
53
Q

3 causes for Down Syndrome

A

1. Nondisjunction (95%): extra chromosome 21 in EVERY cell, due to increased MATERNAL age

  1. Unbalanced translocation (2-3%): all or part of additional chromosome 21 attached to another chromosome
  2. Mosaicism (<2%): the nondisjunction event occured early on and the pt has two disctinct cell lines in their body. The proportion of cells affected are what determines severity of disease
How well did you know this?
1
Not at all
2
3
4
5
Perfectly
54
Q

The relationship between ACh and Alzheimer Disease

A
  • a decrease in ACh level due to a DEFICIENCY of choline acetyltransferase. This is most notable in the basal nucleus of Mynert, which participates in memory and cognition. ALSO involves hippocampus which is obviously involved in new memory formation
  • AChE-I are meds used to treat Alz =Donepezil, Galantamine
How well did you know this?
1
Not at all
2
3
4
5
Perfectly
55
Q

What medication experiences fast and slow acetlyation and happens with what other meds?

A

Isoniazid

  1. Slow acetylation: convert drug slowly and result in an increase in the plasma concentration…also happens with Dapsone, Hydralazine, and Procainamide

2. Fast acetylation:may require higher dose to achieve same therapeutic effect

How well did you know this?
1
Not at all
2
3
4
5
Perfectly
56
Q

The single most effective preventative intervention that pts can do to decrease mortality risk

A
  • STOP SMOKING
  • this is more of an effect than diet or exercise (these effect cardiovascular health but smoking has profound effect on so many different systems in the body)
How well did you know this?
1
Not at all
2
3
4
5
Perfectly
57
Q

Surgery to remove thyroid and now all of a sudden have sx of hypocalcemia, why?

A

Surgery removed parathyroid as well and now the pt is not reabsorbing the calcium in the kidneys anymore

This is called symptomatic hypoparathyroidism due to inadvertant removal of parathyroid

-phosphate reabsorption would also be decreased in these pts

How well did you know this?
1
Not at all
2
3
4
5
Perfectly
58
Q

alpha-galactosidase A deficiency found in what disorder and what are some of the clinical features?

A
  • Fabry, (XL-R) results in buildup of Gb3
  • neuropathic pain, and hypohydrosis are the first sx

Can also see CVA, CVD which are obviously most common cause of death

telangectasia later on

The buildup of Gb3 reuslts in proteinuria and polyuria and eventual RENAL FAILURE

How well did you know this?
1
Not at all
2
3
4
5
Perfectly
59
Q

Fimbriae is another word for ___and is a virulence factor for what?

A

Pilli

E.Coli

How well did you know this?
1
Not at all
2
3
4
5
Perfectly
60
Q

COPD exacerbation tx (not specific med, just mechanism)

A
  • Beta adrenergic agonist (produce relaxation of bronchial smooth muscle by stimulating beta-2 adrenergic receptor)
  • This acts via Gs –> increase cAMP
  • Asthma and COPD are the most frequent causes of pulsus paradoxus in the absence of signficant pericardial disease
How well did you know this?
1
Not at all
2
3
4
5
Perfectly
61
Q

Walk through how a tumor in outer layer of adrenal cortex presents

A

This tumor is affecting the zona gomerulosa which produces aldosterone. Aldosterone acts on the distal collecting duct (principal cells) to increase the absorption of Na and dump out K and H. This if a tumor were producing too much Aldo, you would see a hypokalemic, metabolic alkalosis. These would lead to parasthesias and muscle weakness

This mimics Conn Syndrome

How well did you know this?
1
Not at all
2
3
4
5
Perfectly
62
Q

Where does the clot come from in AFIB?

A

L atrial appendage

How well did you know this?
1
Not at all
2
3
4
5
Perfectly
63
Q

Overactive bladder pathophys and treatment

A

-caused by uninhibited bladder contractions (detrusor instability). Causes a sense of urgency and involuntary leakage

Best treatment is with anticholinergic drugs = oxybutynin, which antagonizes M3 receptors on bladder

How well did you know this?
1
Not at all
2
3
4
5
Perfectly
64
Q

What medications MUST be avoided in a pt with Hypertrophic Cardiomyopathy?

A

Vasodilators: (CCB, nitro, ACE-I) lead to decreased afterload and lower LV volumes

Diuretics: decrease LV filling (preload) and result in greater outflow obstruction

How well did you know this?
1
Not at all
2
3
4
5
Perfectly
65
Q
A
How well did you know this?
1
Not at all
2
3
4
5
Perfectly
66
Q

Actinomycosis

A
  • slow progressive disease caused by the gram + anaerobic bacteria Actinomyces Isralii
  • usually colonize mouth, colon and vagina and can be found in dental caries
  • Most frequently presents with cervicofacial abscesses
  • Pulm actinomycosis is rare but usually occurs because of aspiration.

Dx: filamentous branching, sulfur granules

Tx: PCN

How well did you know this?
1
Not at all
2
3
4
5
Perfectly
67
Q

What property of elastin allows it to stretch during active inspiration and recoil during expiration

A

-Interchin cross-links involving LYSINE made by lysyl oxidase (requires Copper as a cofactor). This forms desmosine cross-links between elastin monomers

Disulfide bridges are made in COLLAGEN NOT ELASTIN

Elastin is made in Alveolar Macrophages

How well did you know this?
1
Not at all
2
3
4
5
Perfectly
68
Q
A
How well did you know this?
1
Not at all
2
3
4
5
Perfectly
69
Q

OSA treatments

A
  • Stimulate hypoglossal nerve (causes tongue to move forward slightly)
  • CPAP
  • surgery
  • wt loss
How well did you know this?
1
Not at all
2
3
4
5
Perfectly
70
Q

Ipatroprium mechanism

A

-anticholinergic agent used for acute asthma attacks that blocks muscarinic receptors (M3), preventing bronchoconstriction

Side effect is increased drying

How well did you know this?
1
Not at all
2
3
4
5
Perfectly
71
Q

Two types of H. Infleunzae

A
  1. Nontypeable are unencapsulated and are the cause of MOST mucosal infections (otitis media, conjunctivitis, bronchitis) 90% of strains isolated from the middle ears of children are this type
  2. H. Influenza Type B (Hib) are responsible for the other 10% of infections. These are capsulated and there is a VACCINE for this type

All children should get this vaccine by 2 months of age to help protect from EMOP (epiglottitis, meningitis, otitis media, and pneumonia)

How well did you know this?
1
Not at all
2
3
4
5
Perfectly
72
Q

What are the neurological symptoms caused by in a panic attack?

A

-Hyperventilation leading to hypocapnia, which can cause decreased cerebral perfusion and altered mental status

How well did you know this?
1
Not at all
2
3
4
5
Perfectly
73
Q

Very high fever, diarrhea, confusion, and a cough that may only be slightly productive is indicative of what??

what is tx?

A
  • Legionnaire’s Disease
  • one of the most common causes of community acquired pneumonia
  • commonly contaminates water sources such as tap water, air conditioners, or other water based cooling systems
    tx: floroquinolones and macrolides
How well did you know this?
1
Not at all
2
3
4
5
Perfectly
74
Q

Lobar Pneumonia stages (4)

A
  1. Congestion (first 24 hours): area is red, heavy, boggy. Alveolar exudate contains mostly bacteria
  2. Red hepatization (days 2-3): Red, firm lobe (liver like consistency) Erythrocytes, neutrophils and fibrin present
  3. Grey Hepatization (days 4-6): Gray-brown firm lobe. RBC disintegrate,
  4. Resolution: restoration of normal architecture
How well did you know this?
1
Not at all
2
3
4
5
Perfectly
75
Q

Dyspnea, Bibasilar Crackles, and presence of S3 are indicative of what??

what does this presence of extra fluid indicate??

A

-Left sided heart failure

-The extra fluid increases resistance to movement and can lead to decreased lung compliance resulting in poor gas exchange and shortness of breath. Other causes of reduced compliance include pulmonary fibrosis and insufficienct surfactant

How well did you know this?
1
Not at all
2
3
4
5
Perfectly
76
Q

SaO2 and PaO2 cutoffs for hypoxic erythrocytosis to begin

A

SaO2 < 92%

PaO2 < 65 mmHG

This is a chronic process, would result in an increase RBC mass (increase retic count) due to increase EPO

How well did you know this?
1
Not at all
2
3
4
5
Perfectly
77
Q

Polycythemia HCT level cutoffs

A
  • Men >52% Hct
  • Women >48% Hct

Then need to differentiate if it is an absolute increase or a relative increase –> need to measure RBC mass (not just Hgb and Hct). If RBC mass is normal, indicates a plasma constriction and relative increase in Hct. If the mass is elevated, must differentiate between Primary and Secondary erythrocytosis by EPO levels (primary have low EPO (myeloproliferative disorder)) (secondary is due to chronic hypoxia from altitude, smoking, COPD)

How well did you know this?
1
Not at all
2
3
4
5
Perfectly
78
Q

What must you tell a pt after inhaled glucocorticoids??

A
  1. use a spacer
  2. Rinse mouth out

If get too much residual glucocorticoid in the mouth can predispose to Candida infection

How well did you know this?
1
Not at all
2
3
4
5
Perfectly
79
Q
A
How well did you know this?
1
Not at all
2
3
4
5
Perfectly
80
Q

What is a CGD pt most susceptible to and why?

A

Most susceptible to Catalase + organisms. CGD is missing NADPH oxidase complex, which produces superoxide, hydroxyl radicals, and hydrogen peroxide.

Pts are susceptible to:

S. Aureus

Burkholderia

Serratia

Nocardia

Aspergillus

How well did you know this?
1
Not at all
2
3
4
5
Perfectly
81
Q
A
How well did you know this?
1
Not at all
2
3
4
5
Perfectly
82
Q

McArdle Disease

A

-Deficiency in myophosphorylase

NO rise in blood lactate levels after exercise

Decrease in this enzyme results in poor exercise tolerance, muscle cramps, rhabdomyolysis

-Can be tolerated by drinking glucose drinks DURING exercise. Not a tx

How well did you know this?
1
Not at all
2
3
4
5
Perfectly
83
Q

why would a newborn with cystic fibrosis present with an intercranial hemorrhage?

A

-They are defienct in fat absorption, thus are defieient in ADEK vitamins, thus are more prone to life-threatening bleeding (esp from the lack of Vit K…making 2,5,7,9,C,S)

How well did you know this?
1
Not at all
2
3
4
5
Perfectly
84
Q

Noise induced hearing loss stems from what?

A

-Trauma to stereociliated hair cells in the organ of corti. High frequency hearing is lost first

Remember:

  1. Hearing frequency at auditory nerve is more likely schwannoma
  2. Cochlear Cupula is responsible for low frequency sounds
  3. Defects of middle ear bones (otosclerosis) affects AIR conduction over ALL frequencies
  4. Malformation of ROUND window is more likely congenital
  5. Rupture of tympanic membrane causes unilateral conductive hearing loss
How well did you know this?
1
Not at all
2
3
4
5
Perfectly
85
Q

Chronic Myeloproliferative Disorders (4)

A

Bone marrow diseases caused by overproduction of myeloid cells

  1. Chronic Myelogenous Leukemia (BCR-ABL 9;22): leukocytosis with LEFT shift
  2. Essential Thrombocytosis (JAK-STAT): thrombocytosis
  3. Polycythemia Vera (JAK-STAT): erythrocytosis

4. Primary Myelofibrosis (JAK-STAT): atypical megakaryotipic hyperplasia, which stimulates fibroblast production which results in progressive replacement of bone marrow by collagen deposition. BONE MARROW FIBROSIS, SEVERE FATIGUE, HEPATOSPLENOMEGALY

How well did you know this?
1
Not at all
2
3
4
5
Perfectly
86
Q

Chromosomal Disorders

3

4

5

7

9

11

13

15

17

18

21

22

X

A

3 VHL, Renal Cell Carcinoma

4: ADPKD (w/ PKD2), Huntington
5: Cru-du-Chat, Familial adenomatous polyposis
7: CF
9: Fredreich Ataxia
11: Wilms tumor
13: Patau, Wilson’s
15: Prader Willi, Angelman
17: NF1
18: Edwards
21: Down
22: NF2, DiGeorge

X: Fragile X, X-linked Aggamaglobulinemia, Klinefelter

How well did you know this?
1
Not at all
2
3
4
5
Perfectly
87
Q

3 AA associated with maple syrup urine disease

(I LOVE VERMONT maple syrup! from maple trees with B1ranches)

A

-Isoleucine, Leucine, Valine

Due to branched chain alpha-ketocacid dehydrogenase complex (BCKDC)

Requires thiamine B1 as a coenzyme**

How well did you know this?
1
Not at all
2
3
4
5
Perfectly
88
Q

Where does H. Pylori live mostly?

A

Antrum of stomach (where there are the fewest acid secreting parietal cells)

How well did you know this?
1
Not at all
2
3
4
5
Perfectly
89
Q

Virulence Factor of Streptoccus Pneumonia

A

polysaccharide capsule (prevents phagocytosis)

  • this capsule is antigenic and Ab are created against it which confers immunity against that particular strain
  • the pneumococcal vaccine confers generates immunity against some common strains (capsule specific)
How well did you know this?
1
Not at all
2
3
4
5
Perfectly
90
Q

Most common immune cells present with Crohn’s

A

Th1

IL-2

INF-Y

Terminal ilium is the most common location of crohns, has cobblestone appearance. Non-caseating granulomas and an inflammatory infiltrate that involves all layers of the wall (transmural) is common

Th2 are more involved in UC!!

How well did you know this?
1
Not at all
2
3
4
5
Perfectly
91
Q

Most cases of hemolytic uremic syndrome are associated with??

A

-E. Coli 0157:H7 due to production of Shiga-like toxin (so obviously can see this in Shigella TOO)

HUS: thrombocytopenia, renal insufficiency, microangiopathic hemolytic anemia

How well did you know this?
1
Not at all
2
3
4
5
Perfectly
92
Q

HPV infections of respitatory tract affect where specifically?

A

-Has a prediliction for stratified squamous epithelium

In the respiratory tract, the only stratified squamous is the TRUE VOCAL FOLDS (think about the constant beating these cells take and they need to be replaced frequently)

False vocal folds, paranasal sinus, trachea: pseudostratified columnar to help protect from foreign bodies

alveoli: simple squamous

“It would truly suck to get HPV in the throat”

How well did you know this?
1
Not at all
2
3
4
5
Perfectly
93
Q

Innervation of ear and area around the ear

A

Majority of area around ear is great auricular nerve (anterior) and great occipital nerve (posterior)

ALTHOUGH, the posterior part of the external auditory canal is innervated by small auricular branch of the VAGUS NERVE

That little bit right inside your ear that is telling you to go to Vegas

How well did you know this?
1
Not at all
2
3
4
5
Perfectly
94
Q

Gram - sepsis caused by___

A

-Release of LPS from bacterial cells during division or bacteriolysis. LPS is NOT ACTIVELY SECRETED

Lipid A is the toxic component of LPS, which causes activation of macrophages leading to widespread release of IL-1 and TNF-alpha

How well did you know this?
1
Not at all
2
3
4
5
Perfectly
95
Q

Low dose vs high dose Dopamine administration

A

Low Dose: stilumates D1 receptors to increase RBF, GFR, and sodium excretion. Mesenteric vasodilation occurs at this level

High Dose: stimulate B1 adrenergic in the heart increaseing pulse pressure, cardiac contractility, and systolic blood pressure

At even higher doses: stimulates Alpha-1 to cause systemic vascoconstriction, which results in decreased CO due to increased afterload (pressor effect)

DBA looks like DoPA or DOBA

How well did you know this?
1
Not at all
2
3
4
5
Perfectly
96
Q

Constrictive pericarditis physical exam findings

A

-Thick fibrous shell that restricts ventricular volumes causes low cardiac output and R sided heart failure

-jugular venous pressure increases (normally this drops during inspiration due to increase in negative pressure) however in these pts, see a paradoxical rise in jvp (KUSSMUAL SIGN)

-Can also hear pericardial knock, which occurs earlier in diastole than S3.

How well did you know this?
1
Not at all
2
3
4
5
Perfectly
97
Q

Giant Cell Arteritis mediator

A
  • IL-6 appears to closely correlate with the severity of the disease
  • Remember that this temporal arteritis is associated with polymyalgia rheumatica and is the most common vasculitis in pts over 50

Cell mediated process with particular increase in

IL-6

Also see increase in ESR

How well did you know this?
1
Not at all
2
3
4
5
Perfectly
98
Q

Most common cause of nephropathy in the US and what is the pathophys and timeline?

A

Diabetic Nephropathy (for both Type I and Type II)

  • The earliest changes are glomerular basement membrane thickening with resultant mesangial matrix expansion. Also see hyaline arteriolosclerosis (in the intima and media of small arteries and arterioles)
  • In initial stages, loss of negative charge of GBM leads to microalbuminemia

Tx: ACE-I have been shown to decrease the protein lost in the urine and slow the progression of GBM thickening

How well did you know this?
1
Not at all
2
3
4
5
Perfectly
99
Q

Loss of sensation in the perineal area?

A

-Cauda equina syndrome, which is commonly due to epidural cord compression from a malignancy. Pts usually develop urinary incontinence late in the disease with fecal incontinence as well

How well did you know this?
1
Not at all
2
3
4
5
Perfectly
100
Q

Major eosinophil functions (2)

A

1. Parasite defense: stimulated by Il-5 (promotes class switching to IgA) via Th2 and mast cells. The parasite invades the system and is coated in IgE and IgG antibodies that bind the Fc receptors of eosinophils. This triggers release of major basic protein that destroys the parasite. This is an example of antibody dependent-cell mediated cytotoxicity

2. Type I hypersensitivity reactions: eosinophils also synthesize prostaglandins, leukotrienes and cytokines that contribute to late phase type 1 hypersensitivity and chronic allergic reactions

How well did you know this?
1
Not at all
2
3
4
5
Perfectly
101
Q

Antiphospholipid Antibody Syndrome

A
  • presence of antiphospholipid antobodies in the setting of thromboembolism and reccurent miscarriages
  • This may be a primary disorder or a disorder secondary to SLE

Remember that SLE can present with a false positive RPR/VDRL test and + antiphospholipid antibodies

How well did you know this?
1
Not at all
2
3
4
5
Perfectly
102
Q
A
How well did you know this?
1
Not at all
2
3
4
5
Perfectly
103
Q

What does angiotensin II do to the efferent arterioles of the kidney?

A

-a POTENT vasoconstrictor that constricts the efferent arteriole and restores GFR

How well did you know this?
1
Not at all
2
3
4
5
Perfectly
104
Q

Which part of the nephron responds to vasopressin??

A

-MEDULLARY segment of the collecting duct and CORTICAL segment

Medullary: production of maximally concentrated urine since this region deals with the HIGHEST osmolarity in the kidney

Cortical: reduces amount of free water delivered to the medulllary collecting duct

How well did you know this?
1
Not at all
2
3
4
5
Perfectly
105
Q

Hypoalbuminemia causes what effect on oncotic pressure?

A

-LOWERS oncotic pressure and causes interstitial edema due to net plasma filtration

How well did you know this?
1
Not at all
2
3
4
5
Perfectly
106
Q

Is H2PO4 increased or decreased in urine of DKA pt?

A

-Increased

This is an acid buffer in the urine that is used to trap free H+ and keep it so the pH does not lower too much

HPO4 and NH3 are the two most important acid buffers in the lumen of the kidney –>they become H2PO4 and NH4

**NH3 is the more important of these two as the kidney can greatly increase NH3 production in chronic acidosis***

How well did you know this?
1
Not at all
2
3
4
5
Perfectly
107
Q

How to calculate Filtration Fraction in kidney?

A

FF=GFR/RPF

This is the fraction of plasma flowing through the glomeruli that is filtered across the glomerular capillaries into Bowman’s Space

RPF can be determined using the PAH clearance as almost ALL of the PAH is excreted in the urine. GFR can be estimated by looking at how much creatinine because this is freely filtered (could use inulin also)

The clearance of anything can be calculated as

Cs=(urine concentration x urine flow rate)/plasma concentration

Note: that 20% is a normal FF for healthy ppl

How well did you know this?
1
Not at all
2
3
4
5
Perfectly
108
Q

Ethylene Glycol effects on kidney

A
  • Leads to TOXIC, acute tubular necrosis with vacuolar degeneration and ballooning of the proximal tubular cells
  • Would find high ANION GAP METABOLIC ACIDOSIS
  • Calcium oxalate crystals in the urine
How well did you know this?
1
Not at all
2
3
4
5
Perfectly
109
Q

What part of the kidney does Lithium act on?

A

-The principle cells of the collecting duct is where lithium can induce diabetes insipitus as a result of lithium’s antagonizing effects on the action of vasopressin

How well did you know this?
1
Not at all
2
3
4
5
Perfectly
110
Q

Spike and Dome appearance of the glomerular basement membrane

A

Seen in Membranous Glomerulopathy

85% of these cases are idiopathic

The remaining cases are due to DM, tumors, SLE, gold, penicillamine, NSAIDS, hep B, hep C, malaria, syphilis

How well did you know this?
1
Not at all
2
3
4
5
Perfectly
111
Q

Chronic Kidney Disease Labs

(GFR, VitD, Phos, Ca, PTH)

A

Starting with a decreased GFR

Decreased 1-25 dihydroxy vitamin D –> decreased intesinal Ca absorption

Phosphate retention –> high serum phosphate

LOW serum Calcium

HIGH PTH synthesis

Secondary HYPERPARATHYROIDISM

How well did you know this?
1
Not at all
2
3
4
5
Perfectly
112
Q

What is the cause of Turner Syndrome

A

-Paternal Meiotic Nondisjunction

The loss of the X-chromosome results in loss of SHOX gene, which is resonsible for long bone growth (reason why pts with Turner have short stature)

How well did you know this?
1
Not at all
2
3
4
5
Perfectly
113
Q

Surgical oophrectomy if the surgeon wants to avoid excessive bleeding need to ligate what??

A

-Suspensory ligament of the ovary

This is what contains the ovarian artery, vein, lymphatics and nerves

**This is the SAME ligament (=infundibulopelvic ligament) that is twisted in ovarian torsion**

How well did you know this?
1
Not at all
2
3
4
5
Perfectly
114
Q

Which of these contains an artery? (esp in regards to removing an ovary)

  1. suspensory ligament of ovary
  2. mesosalpinx
  3. ovarian ligament
  4. Round ligament of the uterus
  5. Transverse cervical ligament (Cardinal ligament)
A
  1. suspensory ligament of ovary –> contains ovarian artery (need to cut for removal of ovary)
  2. mesosalpinx –> region of broad ligament below fallopian tubes and does NOT contain artery
  3. ovarian ligament –> connects ovary to side of uterus (NO blood vessels)
  4. Round ligament –> contains artery of Sampson (not really of consequence when removing ovary)
  5. Transverse cervical ligament –> from cervix to lateral fornix of vagina. Uterine artery does course in superior portion but not relevant for ovarian removal
How well did you know this?
1
Not at all
2
3
4
5
Perfectly
115
Q

Most common cause of septic abortion

A

S. Aureus (gram- bacilli too, like E. Coli and GBS but less common)

Usually an abortion resulting from any retained products of contraception in the uterine cavity

This usually presents as fever, abd pain, uterine tenderness, foul smelling discharge after pregnency termination

How well did you know this?
1
Not at all
2
3
4
5
Perfectly
116
Q

Pt has an enlarged uterus and complains of menorrhagia and dysmenorrhea most likely has what??

A

Adenomyosis which is the presence of endometrial glandular tissue within the myometrium

How well did you know this?
1
Not at all
2
3
4
5
Perfectly
117
Q

Best way to test for trichomonas

A

-Saline microscopy (wet prep) looking for motile protazoan

Usually presents as thin, yellow, mucous discharge that is foul smelling

Tx: metronidazole

How well did you know this?
1
Not at all
2
3
4
5
Perfectly
118
Q

Prior to delivery, estrogen stimulates the upregulation of what inside smooth muscle cells?

A

Gap junctions to heighten myometrial excitation. Gap junctions consist of aggregated connexin proteins that allow passage of ions between myometrial cells

How well did you know this?
1
Not at all
2
3
4
5
Perfectly
119
Q

Gastric tumor that metastasizes to the ovary and what are the pathological findings?

A

Kruckenberg Tumor with classic histologic feature of signet ring cells. These cells appear this way due to a large amount of mucin that displaces the nucleus to the side of the cytoplasm

How well did you know this?
1
Not at all
2
3
4
5
Perfectly
120
Q

Postpardum bleeding can be controlleg by ligating what artery and still keeping fertility possible?

A
  • The uterine arteries are the main blood supply to the uterus and are brachnes off of the internal iliac artery. Bilateral ligation of the internal iliac arteries would stop uterine blood flow and prevent the need for hysterectomy
  • The uterus has collateral blood flow from the ovarian arteries so the blood supply will not be completely cut off
How well did you know this?
1
Not at all
2
3
4
5
Perfectly
121
Q

Episiotomy cuts where and what?

A

This is a cut made from the posterior vaginal opening to the perineal body to enlargen the vaginal outlet to facilitate delivery and reduce the risk for severe perineal laceration

The external anal sphincter, rectal mucosa, levator ani, ischiocavernosus muscle and transverse perineal muscle are NOT cut

How well did you know this?
1
Not at all
2
3
4
5
Perfectly
122
Q

During menstruation what is the process at a cellular level?

A

These cells undergo progesterone withdrawl, leading to a prostaglandin increase, which leads to vasoconstriction of the spiral arteries

The progesterone withdrawl also leads to increased metalloproteases by endometrial stromal cells leading to APOPTOSIS of endometrial epithelium and sloughing of the wall.

How well did you know this?
1
Not at all
2
3
4
5
Perfectly
123
Q

What type of tissue lines the ovary, fallopian tube, uterus, cervix vagina?

A
How well did you know this?
1
Not at all
2
3
4
5
Perfectly
124
Q

Recombination (of viruses)

A
  • exchange of genetic information between two nonsegmented, double stranded DNA genomes. This is the exchange of genes between two chromosomes via crossing over within homolglogous chromosomes
  • Unlike Reassortment, which refers to changes in genomic composition that occur when host is coinfected with 2 SEGMENTED viruses that echange whole genome segments
  • The herpes virus is NOT segmented, so it cannot undergo reassortment
How well did you know this?
1
Not at all
2
3
4
5
Perfectly
125
Q

Cephalosporin-resistant organisms

A

Listeria monocytogenes (resistant PBP)

MRSA (resistant PBP)

Enterococci (resistant PBP)

Atypicals such as mycoplasma and chlamydia (no cell wall)

Treat listeria with ampicillin. Remember, listeria causes disease in those with deficient cell-mediated immunity such as pregnant ladies, young infants, or immuocompromised patients

How well did you know this?
1
Not at all
2
3
4
5
Perfectly
126
Q

Common diseases associated with these:

Borellia Burgdorferi

Pasturella Multocida

Actinomyces israelii

Bacilus anthracis

Rickettsia Rickettsiae

A

Borellia Burgdorferi: Lyme Disease

Pasteurella Multocida: oral flora of cats and dogs

Actinomyces Isralii: oral abscesses with dranining yellow pus

Bacilus Anthracis: cutaneous anthrax. Release Edema factor and Lethal factor creating a papule that eventually is covered with black eschar. Tx: Ciprofloxacin

Rickettsia Rickettsiae: Rocky Mountain Spotted Fever (erythematous macules that migrate centripetally toward the trunk)

How well did you know this?
1
Not at all
2
3
4
5
Perfectly
127
Q

Daptomycin mechanism and uses and adverse effects

A
  • Can be used to treat MRSA (along with vancomycin and linezolid)
  • The mechanism is that it disrupts the bacterial membrane by creating transmembrane channels that cuase intracellular ion leakage. The resulting cellular membrane depolarization and macromolecular (DNA/RNA/protein) synthesis inhibition leads to cell death.
  • It also binds to and is inactivated BY pulmonary surfactant (so it is ineffective in treatment of pneumonia)

Adverse: increased CK levels and increase incidence of myopathy, esp in pts taking statins

How well did you know this?
1
Not at all
2
3
4
5
Perfectly
128
Q

How do you treat recurrence of genital HSV?

A

It can be suppressed or minimized with oral valacyclovir (preffered over valcyclovir as it is dosed once daily and has good bioavailability). Could use daily acyclovir or famcyclovir

-These are not great at being active against latent virus forms, but can suppress multiplication as SOON as reactivation occurs.

A short 7-10 day course of oral acyclovir doesn’t do anything for recurrence rates, it only helps with the lesional healing time and local pain.

How well did you know this?
1
Not at all
2
3
4
5
Perfectly
129
Q

Aspiration Pneumonia vs Aspiration Pneumonitis

A

Aspiration Pneumonia: lung parenchyma INFECTION. Aspiration of upper airway or stomach MICROBES (anaerobes). Presents DAYS after aspiration event. see FEVER, cough, increase sputum. CXR infiltrate in dependent lung segment (classically RLL). Can progress to abscess. ABX: Clindamycin or B-lactam and B-lactamase inhibitor

Aspiration Pneumonitis: Lung parenchyma INFLAMMATION. Aspiration of GASTRIC ACID with direct tissue injury. Presents HOURS after aspiration event. Range from no sx, to nonproductive cough, decreased O2. CXR: infiltrates resolve WITHOUT abx. Tx: supportive

How well did you know this?
1
Not at all
2
3
4
5
Perfectly
130
Q

HSV-1 first clinical sign

A
  • Gingivostomatitis is the first sign and due to its painful nature, the pt opts NOT to drink fluid and often presents to the ER with dehydration
  • Peak age for primary infection is 6months to 5 years
How well did you know this?
1
Not at all
2
3
4
5
Perfectly
131
Q

Most common microorganisms for Cat Bites, Dog Bites, and Human Bites

A

Cats: Pasteurella (most common) has characteristic mouse-like odor, Bartonella (lymphangitis in immunocompromised pts

Dogs: Pasteurella, Streptococci, S. Aureus

Human: Anaerobes, Streptococci, Eikenella corrodens (clenched fist injury “fight bite”)

How well did you know this?
1
Not at all
2
3
4
5
Perfectly
132
Q

Toxic Shock Syndrome Toxin (TSST) mechanism

A

-Acts as a superantigen that activates a large number of helper T cells

This results in release of IL-2 from T cells and IL-1 and TNF from macrophages

This immune cascade is what is responsible for TSST sx

How well did you know this?
1
Not at all
2
3
4
5
Perfectly
133
Q

Q Fever is caused by?

A

Coxiella Burnetti: usually through inhalation of air that has been contaminated by animal waste

Acute: nonspecific febrile illness (HA that are retroorbital, pneumonia, thrombocytopenia)

Chronic: Can be fatal if not treated

How well did you know this?
1
Not at all
2
3
4
5
Perfectly
134
Q

Left shift of oxy hemoglobin dissociation curve has what effect on RBC number?

A

This low oxygen level would stimulate the kidney to produce more EPO, which would result in a compensatory erythrocytosis to maintain normal oxygen delivery

How well did you know this?
1
Not at all
2
3
4
5
Perfectly
135
Q

What do these cells do?

CD4

CD8

CD15

CD16

CD19/CD20/CD21

A

CD4: helper T-cell marker

CD8: cytotoxic T cell

CD15: cell surface marker present on granulocytes. It is also present in nearly all Reed Sternberg cells and is therefore useful in the diagnosis of Hodgkins Lymphoma

CD16: low affinity Fc receptor found on the surfaces of NK cells, macrophages, and neutrophils

CD19/CD20/CD21: found on B-cells (both immature and mature)…would see low in X-linked Agammaglobulinemia

How well did you know this?
1
Not at all
2
3
4
5
Perfectly
136
Q

Cholesteatoma

A
  • Overgrowth of desquamated keratin debris within the middle ear space. May erode ossicles, mastoid air cells and lead to CONDUCTIVE HEARING LOSS
  • Forms a round, pearly mass behind the tympanic membrane
  • Can be primary or secondary to trauma, infection, surgery.
  • Painless
  • Can cause small perforation in the tympanic membrane
How well did you know this?
1
Not at all
2
3
4
5
Perfectly
137
Q

Pt with Hepatic Encephalopathy, what is going on in his astrocytes?

A
  • There is increased levels of ammonia and other toxins circulating around in the body due to an inability to metabolize by the liver.
  • With excess amonia, this is taken up by astrocytes, increasing glutamine production. This leads to increased osmolarity inside of the astrocytes causing swelling and impaired glutamine release
  • Hyperammonemia decreases the amount of glutamine available for conversion of glutamate in neurons resulting in decreased excitation
How well did you know this?
1
Not at all
2
3
4
5
Perfectly
138
Q

How does an RPR test actually work??

A

Serum is mixed with a solution of cardiolipin, lecithin, and cholesterol. If aggregation or “floccation” is seen this is indicative of antibodies against cardiolipin

-This is a non-treponomal specific test, instead it is detecting antobodies to human lipids that are released as a result of cellular destruction by Treponema Pallidum (is positive in yaws, pina, bejel)

How well did you know this?
1
Not at all
2
3
4
5
Perfectly
139
Q
A
How well did you know this?
1
Not at all
2
3
4
5
Perfectly
140
Q

Basic concept of diastolic heart failure (3 things involving EF, EDV, LVP)

A
  • Normal left ventricular ejection fraction (>50%)
  • Normal end diastolic volume
  • Increased LV filling pressure
How well did you know this?
1
Not at all
2
3
4
5
Perfectly
141
Q

Chronic Bronchitis presentation

A

-Thickened bronchial walls with neutrophil infiltration and mucus gland enlargement (increased number of goblet cells) and patchy squamous metaplasia that is brought on by longterm smoking

How well did you know this?
1
Not at all
2
3
4
5
Perfectly
142
Q

Cheyne-Stokes Breathing

A
  • Common in advanced heart failure pts
  • this is a cyclical pattern that has periods of apnea followed by gradually increasing and decreasing tidal volumes that is followed by another apneic period
  • It is also seen in stoke, brain tumors, and TBI’s and is often a poor prognostic sign
How well did you know this?
1
Not at all
2
3
4
5
Perfectly
143
Q

Types of breathing

Normal

Biot

Kussmaul

Cheyne-Stokes

A

Biot-periods of apnea and hyperpnea (only pathological)

How well did you know this?
1
Not at all
2
3
4
5
Perfectly
144
Q

Piriform recess has what function and covers what nerve?

A
  • small cavities that lie on either side of the laryngeal orrifice. During swallowing, food is directed by the eppiglottis to the piroform recess and into the esophagus
  • A thin layer of mucosa protects the internal laryngeal nerve that is a branch of the Superior Laryngeal Nerve (CNX). This contains only sensory and autonomic fibers (NO MOTOR, unlike recurrent laryngeal and external laryngeal)
  • This mediates the afferent limb of the COUGH REFLEX
How well did you know this?
1
Not at all
2
3
4
5
Perfectly
145
Q

Gag Reflex vs Cough Reflex

A

Gag

afferent –> Glossopharyngeal Nerve

efferent –> Vagus nerve

Cough

afferent –> internal laryngeal nerve (branch of superior laryngeal nerve, branch off vagus)

efferent –> vagus

How well did you know this?
1
Not at all
2
3
4
5
Perfectly
146
Q

Treatment of oral Candida infection

A

Nystatin a polyene antifungal with a mechanism similar to amphotericin B. Binds to ergosterol molecules in the fungal membrane causing pores and leakage of fungal cell contents. It is NOT absorbed orally, so is given as an oral “swish and swallow”

could also use fluconazole, or caspofungin

How well did you know this?
1
Not at all
2
3
4
5
Perfectly
147
Q

What makes green color of pus?

A

-Neutrophil Myeloperoxidase a blue-green heme based enzyme that forms hypochlorus acid (bleach)

How well did you know this?
1
Not at all
2
3
4
5
Perfectly
148
Q

CGD is a disease that is missing what enzyme and what are you more prone to as far as infection?

A

Missing NADPH oxidase so more prone to catalase + organisms

C: Candida

A: S. Aureus

T:—-

A: Pseudamonas Aeruginosum

L: Listeria

A: Aspergillus

S: Serratia

E: E. Coli

(and Nocardia)

How well did you know this?
1
Not at all
2
3
4
5
Perfectly
149
Q

What will the pulmonary capillary wedge pressure be in someone in ARDS?

A

-It will be normal because the problem is noncardiogenic pulmonary edema

If the cap pressure was elevated, it is more suggestive of cardiogenic pulmonary edema

In ARDS you WOULD see:

  • increased capillary permeability
  • decreased lung compliance
  • increased work of breathing
  • oxygen diffusion capacity to decrease
  • V/Q mismatch
How well did you know this?
1
Not at all
2
3
4
5
Perfectly
150
Q

Oral thrush, interstitial pneumonia, and lymphopenia in an infant is suggestive of what??

A

HIV infection that was passed mom–>infant

-Could have been avoided with Zidovudine (ZDV) a nucleotide reverse transcriptase inhibitor (enzyme inhibitor) given to the infant for several weeks.

The mom should have had a standard HAART administration of 2 nucleotide/nucleoside reverse transcriptase inhibitors and one third drug (protease inhibitor, NNRTI, integrase inhibitor)

How well did you know this?
1
Not at all
2
3
4
5
Perfectly
151
Q

Theophylline Toxicity

A

adenosine receptor antagonist and phosphodiesterase inhibitor that is used as an alternate therapy for Asthma and COPD. It causes bronchodilation by increasing cAMP levels

-It is metabolized by hepatic cytochrome oxidases, which can be inhibited by medications such as:

-Ciprofloxacin, Cimetidine, Macrolides, Verapamil

Toxicity presents as CNS stimulation such as seizures, tremors, insomnia as well as cardiac abnormalities (arrhythmias)

How well did you know this?
1
Not at all
2
3
4
5
Perfectly
152
Q

Granulomas in TB vs Sarcoidosis

A

TB: caseating granulomas

Sarcoidosis: NON-caseating granulomas

How well did you know this?
1
Not at all
2
3
4
5
Perfectly
153
Q

IL-2 Functions and drug

A

produced primarily by helper T-cells and is a major growth factor for T-lymphocytes

IL-2 also stimulates the growth differentiation and survival of CD4 and CD8 T cells.

**Stimulate the growth of helper, cytotoxic and regulatory T cells and NK cells

IL-2 as a drug is called Aldesleukin and is used for metastatic melanoma and renal cell carcinoma

How well did you know this?
1
Not at all
2
3
4
5
Perfectly
154
Q

Tumor Lysis Syndrome forms what and where (in kidney)

A

Happens when tumors are treated with chemotherapy. The lysis of tumors causes release of K, Phosphorus, Uric acid to be released into the serum

At physiologic pH, the uric acid is soluble, but in the acidic environment of the collecting tubule, they ppt into uric acid stones

This can be prevented by treatment with Allopurinol (Inhibit xanthine oxidase) to prevent tumor lysis syndrome

How well did you know this?
1
Not at all
2
3
4
5
Perfectly
155
Q

Pt with hypertension, hypokalemia, high aldosterone, high renin…how do you differentiate where problem is coming from?

A

Hyperaldosteronism can be divided into primary and secondary

Primary (Conn): There will be hypokalemia, hypernatremia, high aldosterone, LOW renin due to feedback inhibition of renin secretion

Secondary: overproduction of aldosterone is SECONDARY to excess renin production (renal artery stenosis, diuretic use, malignant hypertension, renin-secreting tumor

Nonaldosterone causes: CAH, deoxycortisone producing adrenal tumor, Cushing syndrome, exogenous mineralcorticoids. Would present with LOW aldo, LOW renin

How well did you know this?
1
Not at all
2
3
4
5
Perfectly
156
Q

Pt with MS has what type of bladder abnormalities?

A

-Spastic bladder: urinary frequency and urge incontinence due to the presence of an upper motor neuron lesion (is common in pts developing acute lesions in the spinal cord)

-The bladder does not destend/relax properly due to loss on inhibitory control from UMN

How well did you know this?
1
Not at all
2
3
4
5
Perfectly
157
Q

Multiple myeloma calcium happenings

A

Hypercalcemia due to osteolysis induced by tumor cells, which release osteoclast-activating factor.

This elevated serum Ca leads to decreased PTH production. The low levels of PTH lead to increased loss of Ca in urine (hypercalciuria)

The hypercalcemia and light chain cast nephropathy cause progressive renal failure. This leads to loss of 1-alpha hydroxylase resulting in low 1-25 Vit D.

How well did you know this?
1
Not at all
2
3
4
5
Perfectly
158
Q

Side effects of Foscarnet

A

-hypocalcemia and hypomagnesemia due to it being a pyrophosphate analog. It can chelate calcium

May lead to a reduction in PTH release, which would contribute to the hypocalcemia

Both hypocalcemia and hypomagnesemia can contribute to seizures!

How well did you know this?
1
Not at all
2
3
4
5
Perfectly
159
Q

Acetazolamide use outside of kidney

A

Used in acute angle glaucoma. Carbonic anhydrase modulates HCO3 formation in aqueous humor, so inhibition of CA will decrease aqueous humor production

How well did you know this?
1
Not at all
2
3
4
5
Perfectly
160
Q

Polyuria that responds to DDAVP? and what is the action of DDAVP on the kidney (2 things)

A

Must be central diabetes insipidus (because nephrogenic does not respond at all and central the problem is you just are not producting ADH)

Keep in mind that V2 acts on the kidney to increase aquaporin channels to bring water back in. UREA is also brought back in in the collecting duct in response to V2 in the inner medullary collecting duct, which increases the osmotic gradient in the medulla, which allows for the production of MAXIMALLY concentrated urine

How well did you know this?
1
Not at all
2
3
4
5
Perfectly
161
Q

Label this graph of tubular concentrations as you go along the PCT

A
How well did you know this?
1
Not at all
2
3
4
5
Perfectly
162
Q

What is the origin of Renal Cell Carcioma within the kidney?

and some other fun facts

A

The epithelium of the proximal renal tubules

It looks like polygonal clear cells filled with accumulated lipids and carbohydrates, often golden-yellow due to high lipid content (on macroscopic view)

Risk factors are smoking and obesity

Manifests clinically as hematuria, flank pain, palpable mass (triad that is not always presenting as triad)

Associated with paraneoplastic syndromes (ACTH, EPO, PTHrp, renin)

Tx: Aldesleukin (IL-2)

How well did you know this?
1
Not at all
2
3
4
5
Perfectly
163
Q

Dobutamine

A

B-agonist predominantly B1>B2 that is a simpathomimetic used in heart failure used in management of refractory heart failure associated with LV dysfunction and cardiogenic shock

+inotrope

weakly + chronotrope (this increases HR and thus increases myocardial O2 consumption). Although this can trigger or exacerbate myocardial ischemia, it is weighed against the benefit of end organ perfusion and improvement of cardiac output vs the drawback of cardiogenic shock

How well did you know this?
1
Not at all
2
3
4
5
Perfectly
164
Q

Zenker Diverticulum formation

A

abnormal spasm or deminished relaxation of crycopharyngeal muscles during swallowing is thought to be the mechanism behind Zenker Diverticulum

  • Results in an early oropharyngeal dysphagia that presents as a feeling of food obstruction in the neck area
  • Pts develop halitosis/regurgitation
  • Pulmonary aspiration is common as well leading to pneumonia

NOT A TRUE DIVERTICULUM (not all layers)

How well did you know this?
1
Not at all
2
3
4
5
Perfectly
165
Q

Olanzapine side effects (and Clozapine)

A

Olanzapine=Obesity

This is a second generation antipsychotic. Commonly used first line since it does not have any of the extra-pyramidal side effects that first gen antipsychotics (haloperidol) have.

Olanzapine and Clozapine both have the highest metabolic side effects and it is wise to check fasting glucose and lipid pannel several months down the road when pts are on these drugs

Clozapine can also have agranulocytosis (requires weekly WBC monitoring) Must watch CLOZAPINE CLOZELY

How well did you know this?
1
Not at all
2
3
4
5
Perfectly
166
Q

At what point in the breath cycle is pulmonary vascular resistance LOWEST?

A

-It is lowest at the functional residual capacity (right at the bottom level of the tidal volume)

-Increased lung volumes increase PVR due to longitudinal stretching of the alveolar capillaries by stretching the alveoli

-Decreaed lung volumes increase PVR due to decreased radial traction from adjacent tissues on the large extra-alveolar vessels.

How well did you know this?
1
Not at all
2
3
4
5
Perfectly
167
Q

Give examples of:

Type 1 Hypersensitivity

Type 2 Hypersensitivity

Type 3 Hypersensitivity

Type 4 Hypersensitivity

A

Type 1 (immediate, IgE): anaphalysis, allergies

Type 2 (cytotoxic, IgG and IgM autoantibodies): Autoimmune Hemolytic Anemia, Goodpastures

Type 3 (immune complex, antigen-antibody complex) with complement activation: PSGN, Lupus, Serum Sickness

Type 4 (delayed type, T-cells and macrophages): Contact Dermatitis, TB skin test

How well did you know this?
1
Not at all
2
3
4
5
Perfectly
168
Q

What nerve supplies Deltoid and Teres Minor

A

Axillary, originates from posterior cord and carries fibers from C5 and C6

How well did you know this?
1
Not at all
2
3
4
5
Perfectly
169
Q

Filtration fraction calculation (and normal %)

A

FF=GFR/RPF =20% is normal

RPF=RBF (1-hct)

How well did you know this?
1
Not at all
2
3
4
5
Perfectly
170
Q

Contraindications of buproprion

A
  • Seizure pts
  • Anorexia pts
How well did you know this?
1
Not at all
2
3
4
5
Perfectly
171
Q

Bicuspid Aortic valve risk?

A

Risk of aortic stenosis at age 50!!! due to calcifications and premature atherosclerosis. This is 10-15 years before the normal tricuspid aortic valve starts to see it from normal senile calcific stenosis

How well did you know this?
1
Not at all
2
3
4
5
Perfectly
172
Q

A lesion involving the superior orbital fissure has what effect?

A

-Would eliminate the sensory portion of the corneal reflex that is a branch of the Trigeminal Nerve (the nasociliary branch of the first division of the trigeminal nerve). This enters skull at superior orbital fissure

-Eye adduction via medial recuts muscle that is innervated by the occulomotor nerve (CN3) that enters via superior orbital fissure as well

How well did you know this?
1
Not at all
2
3
4
5
Perfectly
173
Q

Fibromyalgia

A

widespread muscular pain with associated fatigue.

  • effects women 20-55 most commonly
  • Unknown etiology but likely involves abnormal central processing of painful stimuli

Can be exacerbated by excercise although can be aleviated by incremental aerobic exercise.

Can be treated with TCA, SNRI

How well did you know this?
1
Not at all
2
3
4
5
Perfectly
174
Q

4 steps of leukocyte accumulation

A
  1. Margination: increased vascular leakage improves contact of neutrophils with endothelial lining
  2. Rolling: roll on endothelium via lose binding of Siacyl Lewis X to E-selectin, P-selectin on macrophages, L-selectin on neutrophils
  3. Activation: slow rolling allows neutrophils to sample the chemokines associated with the inflammed tissue
  4. Tight adhesion: CD18 to ICAM1
  5. Transmigration: via integrin attachment and adherence to PCAM-1. This protein is found at the peripheral intercellular junction of endothelial cells
How well did you know this?
1
Not at all
2
3
4
5
Perfectly
175
Q

Fibrates (gemfibrozil and fenofibrate) actions

A
  • used to lower triglycerides
  • Activate PAR-alpha which leads to decreased hepatic VLDL production and increased LPL activity. This decreases triglycerides by 25-50% and increases HDL by 5-20%
  • Fish oil also decrease VLDL production and inhibit synthesis of Apolipoprotein B
How well did you know this?
1
Not at all
2
3
4
5
Perfectly
176
Q

Advantages of polysaccharide vaccine for Streptococcus pneumoniae

A

-23 valent polysaccharide vaccine (Pneumovax) protects against a wider range of serotypes but antibody levels decrease over 5 years.

decreased incidence of replacement strains due to lack of mucosal immunity

How well did you know this?
1
Not at all
2
3
4
5
Perfectly
177
Q

Advantages of the conjugate vaccine for Streptococcus pneumoniae

A

-13 valent (Prevnar) contains a nontoxic diptheria protein conjugated to the polysaccharide that boosts the immune system through T-cell recruitment

Increased efficacy in the elderly and the young < 2

-Increased mucosal immunity reduces colonization (herd protection)

-Increased immunogenic memory (strongly immunogenic)

178
Q

Label the different parts of an antibody and what binds where?

A/B

C

D

E

A

A/B. Site for attachment of antigen in FAB. This is the hypervariable region

C. The two disulfide bonds that hold the heavy chains together

D. C1 must bind FC portions of IgM and IgG in order to activate the classical complement pathway.

E. The Fc binding part of the constant region of the heavy chain. This is the site for attachment to phagocytitic cells. MACROPHAGES. MAST cells can bind the FC portion of IgE

179
Q

Anicertic

A

Hepatitis without jaundice (without sclera icterus =icteric)

In children <6 with HAV, 80% will be anicteric

In adults, clinical disease is more severe with 70% of symptomatic patients presenting with jaundice

HAV is a self limiting virus that is NOT associated with a carrier state

180
Q

NK cells express which CD?

A

CD16, CD56

DO NOT express CD4, CD8, CD3

DO NOT require thymus for maturation

DO NOT have antigen specific properties, thus

DO NOT require exposure to antigen for activation thus

DO NOT possess antigen memory ability

…activated by IL-12, INFY

181
Q

What molecule is involved in mediating the accumulation of pus

A

Pus consists of a thin protein rich fluid and dead leukocytes, predominently neutrophils

During infection, macrophages and endothelial cells release cytokines such as IL-8 that trigger neutrophils to enter the site of infection via chemostaxis

Clean up on aisle 8” Neutrophils recruited by IL-8 to clear infections

182
Q

Mast Cell fun facts

A

mediates allergic reaction in local tissue

Bind the Fc portion of IgE molecules

IgE crosslinks upon antigen binding –> degranulation –> release of HISTAMINE, HEPARIN, TRYPTASE (can be used as a marker of mast cell activation)

-Chromolyn sodium prevents mast cell deganulation

183
Q

Mast cell receptor aggregation

A

The high affinity IgE receptor is found on the surface of mast cells. The IgE receptor binds the FC of the IgE molecule. When the antigen binds to the FAB region of the IgE, multiple IgE antibodies become crosslinked resulting in aggregation of receptors on the mast cell surface

This results in activation of non-receptor tyrosine kinases that triggers an intracellular cascade that results in mast cell and basophil degranulation

184
Q

IgA protease, who produces it and what the heck does it do?

A

Produced by Neisseria Gonorrhea, Neisseria Meningitidis, Strep Pneumo, H. Infleunza

This protease cleaves IgA at the hinge region yielding FAB and compromised Fc fragments, thus decreasing its effectiveness

This facilitates bacterial adhearance to mucosal surfaces

185
Q

Very low immunoglobulin levels is suggestive of what?? and what does it present with in lymph node?

A

Bruton Agammaglobulinemia

NO B cell maturation in bone marrow. Get buildup of pre-B Cells in bone marrow

Caused by mutation in Bruton Tyrosine Kinase gene

-B lymphocytes normally in the cortex of lymph nodes to form lymphoid follicles

Primary: dense and dormant

Secondary: pale and contain proliferating B-cells

A pt with Bruton would LACK GERMINAL CENTERS where B-cells normally hang out

186
Q

Tetanus is prevented by??

A

Tetanus toxoid vaccination (formaldehyde inactivated tetanus toxin). The vaccination initiates humoral immunity specific to the tetanus TOXIN.

-The tetanus immune globulin can be administered during the acute treatment of tetanus or for tetanus prophylaxis after an injury

**Remember, tetanus disease is caused by the TOXIN, not the circulating Clostridium Tetani, thus regular host defense mechanisms are NOT effective against tetanus. REASON WHY YOU NEED TO BE VACCINATED

187
Q

Number needed to treat

A

1/ARR (absolute risk reduction)

where ARR is the difference of the treatment vs the placebo group.

188
Q

3 meds that can be used for C. Diff tx

A

-Vancomycin (ORAL)

  • Metronidazole
  • Fidaxomicin: a macrolide antibiotic that that inhibits the sigma subunit of RNA polymerase leading to protein synthesis impairment (this is bacteriocidal). Particularily useful for recurrent C. Diff. It has a lower recurrence rate and higher cure rate
189
Q

What muscle passes through the greater sciatic foramen?

A

-Piriformis muscle: involved in external hip rotation

190
Q

Where in the cell cycle do these act:

Vinca alkaloids (Vinblastine, Vincristine)

Bleomycin

Doxirubicin

Antimetabolites (5FU, methotrexate, hydroxyurea, 6-mercaptopurine)

Cysplatin, Alkalating Agents

A

Vinca alkaloids: M phase

Bleomycin: G2 phase

Doxirubicin: G2 phase

Antimetabolites: S Phase

Cysplatin, Alkalating Agents: G0, cell cycle independent

191
Q

Dillusion Disorder vs Schizophrenia

A

Dilusion Disorder: >1 dilusion >1 month, other psychotic symptoms NOT present (NO hallucinations, NO disorganization). Typically have a persistent overridding delusion with a specific theme (genuinely believe you are married to a celebrity, genuinely believe being cheated, spied on)…must be somewhat realistic)

Schizophrenia: periods of psychosis and disturbed behavior and thought lasting >6 months. Need >=2 of the following: dillusions, hallucinations, disorganized speech, disorganized or catatonic behavior, negative sx (flat affect, anhedonia)

192
Q

E. Coli virulence factors for:

bacteremia, septic shock

Neonatal Meningitis

Gastroenteritis (bloody)

Gastroenteritis (watery)

UTI

A

Bacteremia, septic shock –> lipopolysaccharide

Neonatal meningitis –> K1 capsular polysaccharide

Gastroenteritis (bloody) –> Shiga-like toxin

Gastroenteritis (watery) –> Heat labile (cAMP), heat stable (cGMP)enterotoxins

UTI –> P fibriae

193
Q

Alcohol Withdrawl Progression

A

mild withdrawal –> seizures –> hallucinations –> DT

194
Q

Narcolepsy tx

A

Daytime: Modafinil, Amphetamines

Nighttime: Sodium oxybate

195
Q

Pts with diabetic nephropathy are often started on what medicine ___ and what is a potential side effect ____ and how to avoid it

A

These pts are started on ACE-I because the diabetic neuropathy stems from the nonenzyamatic glycosylation of the GBM that is favoring the efferent >afferent arteriole. This leads to increased GFR and eventual microalbuminURIA. This can be delayed with an ACE-I

  • When starting ACE-I, it is common to see first dose hypotension (espectially when the pt is already on a diuretic and has high renin levels). The ACE-I causes an abrupt removal of the vasoconstrictive effects of ANG II resulting in decreased peripheral vascular resistance and a precipitous drop in BP
  • This can be avoided by giving small doses and incrementally increasing
196
Q

What is the effect of PaO2 with carbon monoxide poisoning

A

THERE IS NO EFFECT!!

PaO2 is the amount of O2 DISSOLVED in plasma and represents the partial pressure of oxygen.

What is decreased is the amount of hemoglobin that are bound to O2. This will decrease as CO has a higher affinity for Hb

197
Q

Thayer Martin Agar is made up of what and grows what?

A

Vancomycin –> inhibit gram + bacteria

Polymyxin –> inhibit gram - bacteria

Nystatin –> inhibit yeast

Trimethoprim –> inhibit proteus

Used to isolate....Neiseria

198
Q

Fragile-X syndrome findings…

A

Neurobehavioral problems

LONG face

Prominent forehead

Large Ears

Prominent chin

199
Q

Amphotericin B mechanism and use

A

-Binds ergosterol (unique to fungi); forms membrane pores that allow leakage of electrolytes

“AmphoTERicin TEARS holes in fungal membrane”

Use for serious systemic mycoses:

-Cryptoccus

Coccidiode

-Histoplasmosa

-Candida

-Mucor

Supplement K and Mg because of altered renal tubule permeability

-Side efects: F/C. hypotension, nephrotoxicity, arrhythmias, anemia, IV phlebitis

*Hydration decreases nephrotoxicity.

200
Q

Physical location of systyemic mycoses:

Mississippi and Ohio River Valley

Eastern US and Central America (great lakes)

Southwestern US, California

Latin America

A

Mississippi and Ohio River Valley –> Histoplasmosis

Eastern US and Central America (great lakes)–> Blastomycosis

Southwestern US, California–> Coccidioidomycosis

Latin America–> Paracoccidioidomycosis

201
Q

Size of systemic mycoses in relation to RBC and one definining feature:

Histoplasmosis

Blastomycosis

Coccidiodomycosis

Paracoccidioidomycosis

A

Histoplasmosis –> smaller than RBC (macrophage filled with histo)

Blastomycosis –> same size as RBC (broad-based budding)

Coccidiodomycosis –> much larger than RBC (Spheru;e filled with endospores)

Paracoccidioidomycosis –>MUCH larger than RBC (Captain Wheel)

202
Q

Celiac Disease pathology findings

A
  • Villous Atrophy
  • Crypt Hyperplasia
  • Intraepithelial lymphocyte infiltration

Dx: anti-tissue trans-glutaminase (anti-TTG), IgA anti-endomysial and is confirmed by tissue biopsy

Tx: with a gluten free diet

203
Q

RPF = what?

A

RPF= PAH clearance (remember this is fully secreted in PCT, not reabsorbed at all, but is also not filtered at all at the glomerulus, so is a perfect number to use for RPF!!!!)

RPF=PAH clearance =[(urine PAH x urine flow)/(plasma PAH)]

Then remember that RBF=(PAH clearance)/(1-Hct)

204
Q

Which polymerase is constrained primarily to the nucleolus??

A

RNA Polymerase 1 functions exclusively within the nucleolus (the site of ribosomal maturation and assembly) to transcribe the 45s pre-RNA gene, which codes for MOST of the RNA components

205
Q

Phenotypic Mixing

A

Occurs with simultaneous infection of a cell with 2 viruses and results in progeny virus that has nucleocapsid components of A and B

*The key here is that there is no exchange of genomic viral genomes and thus in the subsequent generations, A –> A and B –> B

206
Q

Patient cannot read, what do you do??

A

Use visual aids to aid in explaination

DO NOT make the pt read something, may be embarassing and unnecessary

207
Q

Dizygotic Twins

A

Fertilizaiton by two sperm of two eggs and ALWAYS have 2 amnions and 2 chorions

208
Q
A

A/B. Phosphoglucomutase (interconverts Glucose-1-phosphate and Glucose-6-phosphate

C. Glucose-6-phosphate dehydrogenase (G6PD)

D. Glucose-6-phosphatase (McArdle)

E. Hexokinase/Glucokinase (hemolytic anemia, rare)

F/G Phosphoglucose Isomerase (hemolytic anemia)

209
Q

Ionoropic vs metabotropic receptors

A

Ionotropic: ion channels that open directly upon ligand binding. NICOTINIC…allow immediate influx of ions, such as a nonselective cation channels that open after the binding of ACh.

Metabotropic: use second messengers. MUSCARINIC

210
Q

4 year old boy failure to thrive, excercise intolerance, SOB, palpable thrill over L sternal border with continuous murmur. What is dx?? and what is embryological origin

A

Patent ductus arteriosus, an embryonic development of the 6th aortic arch that allows blood to pass from the pulmonary artery to the descending aorta and bypass the lungs and left side of the heart.

  • The vessel closes with the decrease of prostaglandin E2 and increased oxygen concentration.
  • If patent after birth, results in a L –> R shunt that can cause left ventricular volume overload and symptoms of heart failure
  • Can close pharmalogically by indomethacin (PGE2 inhibitor)
211
Q

treatment of status epilepticus

A

IV benzodiazepam (lorazepam, diazepam)

Benzos enhance GABA-A causing influx of negatively charged Cl ions in response to GABA binding. This leads to hyperpolarization of the post-synaptic neuron and suppression of AP firing (anticonvulsant effect)

Status epilepticus is reccurent seizure or seuzire lasting >5min

212
Q

epithelial ovarian cancer protective meausres

A

OCP

Multiparidy

Breastfeeding

Salpingo–oophrectomy

-Cancer antigen-125 is a protein produced by ovarian epithelium and is usually increased in cancerous ovarian cells compared to normal cells (not sensitive or specific)

213
Q

idiopathic intracranial hypertension (pseudotumor cerebri)

A

Increased ICP with no apparent cause

  • Young woman of childbearing age w/ daily headache. bilateral symmetrical papilledema, transient vision change, and sx worsen when bending down
  • Compression of optic nerve externally that impairs axoplasmic flow within optic nerve
    tx: weight loss, acetazolamide, topiramate (seizure)
214
Q

Propofol distrubution inside the body

A
  • Potentiates GABAa
  • A highly lipophilic drug that is used for both induction and maintainence of general anesthesia as well as procedural sedation
  • Onset of action is 30 sec, and duration is <10 min. It goes where blood flow is highest (brain) first then goes to areas where blood flow is lower such as fat, muscle tissue
215
Q

Cerebral Amyloid Angiopathy

A

Can cause recurrent lobar hemorrhage is a consequence of B-amyloid deposition in the walls of small–> medium sized cerebral arteries resulting in vessel wall leakage and predisposition to rupture

*The most common cause of spontaneous lobar hemorrhage in adults >60 and tends to involve the parietal and occipital lobes

216
Q

Characteristics of Aortic Regurgitation

A

Early diastolic murmur

  • Signs and symptoms of left heart failure are common
  • more advanced cases are a more holosystolic murmur while earlier cases are associated with an early diastolic murmur
  • MOST commonly caused by aortic root dilation or bicuspid aortic valve
217
Q

Germinal Matrix Hemorrhage

A
  • usually results in intraventricular hemorrhage (most common in infants born before 32 weeks gestation and <1500 grams), and almost always occurs in first 5 days postnatal
  • The germinal matrix is a highly vascular area where neurons and glial cells migrate out of during development. There are many thin walled vessels in this area that lack the glial fibers and support that other vessels have in the brain
218
Q

Name the insulin lines

A

A=Aspart (first)

219
Q

Why does lactic acidosis cause anion gap metabolic acidosis?

A

-Lactic acidosis is produced by over production or underexcretion of lactic acid. End organ hypoperfusion in septic shock impairs tissue oxygenation and decreases oxidative phosphorylation leading to a buildup of NADH and shunting of pyruvate to lactate following glycolysis

220
Q

Pt is in anaphylactic shock! what med do you give them??

A

-EPINEPHRINE!!

This is the drug of choice due to its ability to reverse all of the pathophysiologic mechanisms of anaphylaxis.

  1. Stimulation of Alpha-1 counteracts the vasodilation of cutaneous and viscera vasculature.
  2. Stim B1 to increase contractility and CO to increase BP and improve peripheral circulation
  3. Stim B2 to result in bronchodilation

Diphenhydramine may be used AFTER stabilization of the pt, but is NOT a first line med!

221
Q

Pterion fracture (skull)

A
  • This is the spot where the frontal, parietal, temporal and sphenoid bone all join.
  • A fracture here is common to cause a middle meningial artery laceration.

The middle meningial artery is a branch off the maxillary artery

-This rupture of an ARTERY –> epidural hematoma, which can lead to increased ICP and cushing reflex, brain herniation, and death

222
Q

Caudal Regression Syndrome

A

Pts born with agenesis of the sacrum, and lumbar spine area and experience flaccid paralysis of the legs, and urinary incontinence

-It is frequently related to maternal diabetes that were poorly controlled though there are other factors likely at play

223
Q

Telomerase what does it do and what cells is it found on?

A

-ribonucleoprotein that adds TTAGGG repeats to 3’ end of chromosome (telomere region)

It synthesizes single stranded DNA using single stranded RNA as a template (RNA dependent DNA Polymerase)

Stem cells have very long telomeres due to their high telomerase activity

Myocardial cells, neurons, pancreatic beta cells would be examples of cells that have SHORT telomeres as they fully differentiated cells

224
Q

Hemolytic Uremic Syndrome is caused by what?? OK, and what is the actual mechanism that we see?

A

Caused by E. Coli O157:H7 or Shigella Dystentarie

The Shiga toxin enters the circulation and causes capillary endothelial damage, resulting in platelet activation and microthrombi. This results in thrombocytopenia due to the platelet consumption and schistocytes from the RBCs damaged during the microthrombi resulting in hemolytic anemia

The hemolytic anemia causes decreaed hemoglobin and haptoglobin in the blood as well as increased serum lactate and unconjugated billurubin

225
Q

Flecainaide

A

Class 1C antiarrhythmic that is typically used to treat SVT caused by AFIB

These bind to the Na channels responsible for phase 0 depolarization and prolong QRS duration

Class 1C drugs are the slowest to dissociate from the Na channels resulting in a “use dependence” in which their sodium blocking effect is increased when the HR is increased

226
Q

Dofetilide

A
  • A class III antiarrhythmic that blocks outward potassium current during repolarization. As a result, it increases the QT but NOT the QRS
  • This expresses reverse use dependence, the lower the HR, the more effect it has
227
Q

Central Scotomas

A

-The macula is a dense bit of cones located 1.5 mm near the retina

Macular lesions impair central vision and result in CENTRAL SCOTOMA

Scotoma = a visual defect surrounded by regular vision

228
Q

Ataxia Telangectasia mutation

A

ATM gene responsible for DNA break repair –> cell cycle arrest

-Mutation in this gene is AR and results in cerebellar ataxia, telangectasias and increased risk of sinopulmonary infections

Usually have an IgA deficiency as well

Can see an increased AFP in these pts too

229
Q

Collagen Synthesis mechanism

A

Begins with the transcription of collagen in the nucleus. Alpha chains are then synthesized by the RER bound ribosomes and directed into the cisternae of the RER.

Within the RER, proline and Lysine are post-translationally hydroxylated to hydroxylysine and hydroxyproline via prolyl hydroxylase and lysyl hydroxylase.

Defective hydroxylation severely diminishes the amount of collagen secreted by fibroblasts and impairs triple helix production. VITAMIN C is required for HYDROXYLATION

230
Q

Fun facts about Parvovirus

A

Parvovirus B19 is a single stranded DNA naked virus that is also called 5th disease (think getting 5 fingers across face)

  • Causes classic slapped cheek look on kids that develops after the nonspecific prodrome sx resolve (fever, malaise, congestion, HA). The rash may be a result of immune complex deposition (IgM, IgG)
  • Parvovirus replicates in ERYTHROCYTE PRECURSORS of bone marrow. Can see aplastic crisis in pts with sickle cell disease
231
Q

Measles Virus progression

A

Rubeola:

Prodrome symptoms of: Cough, Coryza, Conjunctivitis, fever, and Koplik Spots on the buccal mucosa (these are pathognomonic for Measles

These symptoms precede a maculopapular exanthem that starts on the face and spreads downward

232
Q

Neuroleptic Malignant Syndrome

A

life threatening reaction to anti-psychotic medications, which block dopamine receptors in the brain

Presents clinically with diffuse muscle rigidity, high fever, autonomic instability (HTN, tachycardia), and altered sensorium.

CK through the roof as pt has rhabdo

**Think LEAD PIPE RIGIDITY FOR NMS*

*Think hyperreflexia for serotonin syndrome

Tx: discontinue psych med, DANTROLENE (blocks release of Ca from sarcoplasmic reticulum via antagonizing ryanodine receptors)

233
Q

Trendelenburg Gate is caused by what? and where is the proper spot for an injection (because improper injection technique can cause this problem in the first place…spoiler alert)?

A

Trendelenburg gate is the dropping of the hip on the contralateral side that is affected. On the affected side, the superior gluteal nerve is affected.

This can be a result of a misplaced injection.

The proper site for an injection is superiorlateral quadrant at a 90 degree entrance is correct.

Injections into superiormedial –> risk superior gluteal

Injections into superiormedial, inferiormedial, inferiorlateral are ALL increased risk of –> sciatic nerve involvement

234
Q

Paroxysmal Nocturnal Hemoglobinuria (PNH)

A

Hemolytic Anemia, Hypercoagulability, and Pancytopenia is the triad of the disorder.

The problem is acquired mutation in PIGA gene that causes absence of GPI anchor that attach CD55 and CD59. These proteins help to inactivate complement and prevent MAC from activating.

Since they are not functioning, complement is activated too much and normal cells are broken down via a complement-mediated-hemolysis

-Pts can also develop Budd-Chiarii Syndrome due to release of free hemoglobin and prothrombotic factors from lysed cells

235
Q

Why is essential fructosuria a benign condition?

A

Because hexokinase can take over and convert frucose –> fructose-6-phoshate instead of the fructokinase that normally converts fructose –> fructose-1-phosphate

This is NOT like Hereditary fructose intolerance where you are deficient in Aldolase B. At this point, you have already converted to Fructose-1-phosphate and are STUCK and cannot convert back via hexokinase to Fructose-6-phosphate

236
Q

PPO vs HMO vs POS for insurance options basic overiview

A
237
Q

S-100 + stain

A

Neural Crest Origin

Schwannomas 100%

Neurofibromas 100%

Ependamomas

Astrogliomas

Melanomas

Langerhan Cells

238
Q

What are the encapsulated bacteria and what is a good mnemonic for them? And what questions can you answer just from this info?

“Some nasty killers have some capsule protection”

A

“Some Nasty Killers Have Some Capsule Protection”

Strep Pneumo

Neisseria Meningitis

Klebsiella

Haemophilus Influenza

Salmonella Typhi

Cryptococcus Neoformans

Pseudamonas Auerginosa

-Keep in mind these would all be difficult for an asplenic pt, IgG def, C3 def, Bruton’s. Capsule is usually major virulence factor, and if + Quellung, probably one of these bugs

Big mucoid colonies on blood agar is Klebsiella

239
Q

Explain each layer and which layer specifically is affected by Pernicious anemia and what is pernicious anemia

A

Pernicious anemia is an autoimmune disorder caused by CD4+ cells against parietal cells in the gastric body and fundus. Over time, this results in atrophic gastritis

A: simple columnar epithelium: secretes mucous

B: Upper glandular layer: where parietal cells secrete gastric acid and intrinsic factor

C: Deeper aspect of gastric glands: This is where Chief cells hang out and secrete pepsinogen

D: Muscularis Mucosa: separates lamina propria from submucosa

E: Submucosa: Where CT is

240
Q

What are chemotactic for neutrophils (4)

A

Leukotriene B4

IL-8

C5a

Bacterial products

241
Q

Early and late onset Alzheimer Disease is brought on by what mutations

A

Early: APP (chr21), presenilin 1, presenilin 2

Late: apoE4

242
Q

Treatment of Myasthinia Gravis is what? what are the side effects and what is a drug used to treat these side effects??

A

Tx w/ cholinesterase inhibitor such as pyridostigmine. Side effects are excessive cholinergic stimulation of the gut leading to abd cramping, sweating, diarrhea, nausea.

These symptoms can be treated with Scopolamine, a selective muscarinic ACh antagonist that reduces the effect of cholinesterase inhibitors

(also used in motion sickness, which presents with very similar sx)

243
Q

Closed head injury pt. Hyperventilate or hypoventilate and why?

A

Hyperventilate to bring down the pCO2. This causes vasoconstriction and reduction in cerebral blood flow resulting in decreased ICP

*Think that the ICP is already raised from edema, so trying to do anything to reduce this swelling*

244
Q

Biotin (B7) is a cofactor for what enzymatic reactions?

A

Important for several carboxylase enzymes

Deficiency of biotin can arise from poor diet, excessive raw egg white consumption (due to high levels of biotin-binding avidin in egg whites)

1. Pyruvate carboxylase

2. Acetyl CoA carboxylase

3. Proponyl CoA carboxylase

Present with non-specific symptoms but can include dermatologic changes* mental status change, anorexia

245
Q

Gingival Hyperplasia is caused by what drug?

A

Phenytoin! due to increased expression of PDGF

It occurs in 50% of pts who have been on phenytoin for 3-4 months or longer

(also cyclosporine and tacrolimus –> calcineurin inhibitor, as well as CCB)

246
Q

Label parts of brain…

A

A. Caudate (A/C are striatum together)

B. Internal Capsule

C. Insular Cortex (limbic system)

D. Putamen

E. Globus Pallidus

(D/E make up Lentiform nucleus)

247
Q

What med can be used to treat alcoholism that is NOT antabuse…

A

Naltrexone: blocks the mu-opioid receptor. It blocks the rewarding and reinforcing effects of alcohol and has been shown to reduce the craving of alcohol

As a reminder, Disulfuram (antabuse) does inhibit aldehyde dehydrogenase and causes accumulation of acetaldehyde and causes the unpleasant hangover-like symptoms

248
Q

Treatment of Diptheriae (in order)

A

1. Diptheria Antitoxin (inactivates all circulating toxin, but is ineffective against toxin that has already gained access to cardiac or neural cells). This process is considered PASSIVE IMMUNIZATION, as it represents the transfer of pre-existing, neutralizing antibodies

2. PCN or erythromycin kills bacteria and halts making of toxin

3. DPT vaccine (active immunization) protects against future infections

249
Q

Mechanism of opiates

A

-Blocks Mu-receptor via binding to Gi proteins. On presynaptic side: results in closure of voltage-gated calcium channels, reduced calcium influx and thus decreased excitatory NT release.

On post-synaptic side: opens potassium channels and allows for K efflux from cells leading to membrane hyperpolarizaiton

250
Q

What arteries have the highest burden of atherosclerosis

A

Lower abdominal aorta –> Coronary Arteries –> carotids –> popliteal

251
Q

Subacute Sclerosing Panencephalitis

A

Rare complication of measles (rubeola) that occurs years after initial infection.

Presents with personality changes –> lethargy –> difficulty in school/odd behavior –> dementia –> myoclonic jerking –> flaccidity + decorticate rigidity.

Rembmer, the measles is a paramyxovirus that presents with cough, coryza, conjunctivitis and Koplik spots that progress to maculopapular rash that starts at the head and spreads downward

252
Q
A
253
Q

Mood stabilizers in bipolar disorder (4) and their side effects

A
  1. Lithium: hypothyroidism, DM, ebstein anomaly, tremor
  2. Valproate: HEPATOTOXICITY, neural tube defects
  3. Carbamezapine: AGRANULOCYTOSIS, hyponatremia, NTD
  4. Lamotrigine: SJS
254
Q

Enteropeptidase function

A

activates trypsinogen to trypsin

Trypsin can then go ahead and activate all the other pancreatic enzymes. Trypsin can also begin to break down proteins to oligopeptides and amino acids

Enteropeptidase hangs out in the brush border of the duodenum

255
Q

Folate pathway and folate deficiency can be shortcutted by what?

A

Can be shorcutted by thymidine supplementation. This converts dUMP to dTMP

256
Q

DSM5 Personality disorders

paranoid

schitzoid

schitotypal

antisocial

borderline

histrionic

narcissistic

avoident

dependent

obsessive compulsive

A
257
Q

In regards to testicular arteries and veins, two come off what and 1 and 1 come off what?

A

R testicular vein: inferior vena cava

L testicular artery: abdominal aorta

R testicular artery: abdominal aorta

L testicular vein: L RENAL VEIN reason why there is increased risk of varicocele on the left side

258
Q

Paraneoplastic syndrome that affects cerebellum

A

seen in small cell lung cancer, breast, ovarian, and uterine cancer

Anti-YO, anti P/Q and anti-HU antibodies

pts present with worsening dizziness, atazia, and visual disturbances

These cross react with Purkinje neuron antigens leading to acute onset degeneration of the cerebellum

259
Q

label the parts of the brachial plexus

A
260
Q

refamiliarize yourself with the coronary arteries, especially in a R dominant coronary artery person in regards to papillary muscle rupture after an MI

A

*the posteriormedial papillary muscle is supplied solely by the posterior descending artery unlike the anteriormedial papillary muscle that is supplied by a duel blood supply of the LAD and the L circumflex

papillary muscle rupture is a life-threatening condition that happens 3-5 days after a myocardial infarction and presents with acute mitral regurgitation and pulmonary edema

261
Q

Immunoflourescence of PSGN shows what?

A

IF shows granular “lumpy bumpy” (not linear like seen on goodpastures syndrome)

What is actually showing up is C3b, IgG, and IgM in the mesangium and basement membranes

262
Q

CD14

A

monocyte/macrophage lineage

263
Q

esophageal manometry for achalasia, scleroderma and diffuse esophageal spasm

A

scleroderma has high pressure at prox esophagus

achalsia is high LES tone

264
Q

what is lymph node drainage around genital area?

Think about where the testes came from and where the penis goes and what the scrotum is attached to

A

Scrotum –> superficial inguinal lymph nodes. These lymph nodes drain nearly all cutaneous lymph from the umbilicus to the feet including the external genitalia and anus (up to the dentate line)

THE EXCEPTION is the testes, glans penis, and cutaneous portion of posterior calf

Lymph from the testes –> para-aortic (retroperitoneal)

Glans penis and posterior calf –> deep inguinal lymph nodes

265
Q

What is this picture of and what is the most likely origin?

48 year old female, weakness shoulder and hip girdle, minor abd pain

A

The picture is of Dermatomyositis, an autoimmune disease characterized by proximal muscle weakness resembling poliomyolitis.

findings include heliotrope rash, Gottron pappules (pictured on hands)

-This condition may occur alone or as part of a paraneoplastic syndrome from ovarian adenocarcinoma (but can also be lung, colorecta, non-hodgkins)

Anti-jo, Anti-SRP, anti-Mi-2

266
Q

Niacin side effect and mechanism in regards to cholesterol treatment

A

Used as treatment for hyperlipidemia to inhibit hormone sensitive lipase and reduce hepatic VLDL synthesis

Side effect include flushing, warmth and itching

This is mediated by PROSTAGLANDINS (PGD2 and PGE2)

ASA can decrease this side effect if given 30 min prior to administration of niacin

267
Q

pt trying to commit suicide eats two bottles of insecticide. What is the poisoning we would see and what is the tx?

A

Arsenic poisoning.

Binds to sulfhydryl groups, impairing cellular respiration via inhibiton of pyruvate dehydrogenase and disruption of gluconeogenesis

GARLIC odor is often found on pt breath. Abd pain, diarrhea (watery), delerium, and hypotension are also seen.

Tx: dimercaprol (chelating agent)

268
Q

Osler-Weber-Rendu syndrome

A

AD inheritence of congenital telangectasias to the skin

ALSO called hereditary hemorrhagic telangectasia

presents with recurrent nose bleeds, skin discolorations

269
Q

Theophylline mechanism and overdose sx

A

used in asthma. A phosphodiesterase inhibitor that likely causes bronchodilation by increasing cAMP

Side effects: (think about it having the same mechaism as caffeine) at low to moderate doses, can cause mild cortical arousal and insomina. Acute intox can cause nausea/vomiting, diarrhea, cardiac arrhythmias and seizures. Seizures are the major cause of morbidity

270
Q

accuracy, precision, reliability, and validity

A

Reliability (precision) measure of statistical variation. A reliable test is reproducible but not necessarily giving the right answer (bottom two on figure)

Validity (accuracy) the tests ability to measure what it is supposed to measure

271
Q

How long do you need depressive symptoms to be considered a major depressive disorder

A

> 2 weeks and need >5 of the 9 possible symptoms, with no hx of mania or hypomania and no curent substance abuse that could be causing problem

272
Q

microsomal monooxygenase

A

reaction associated with CYP450 system

some pro-carcinogens are metabolized by cytochrome P450 microsomal monoxygenase

273
Q

P bodies

A

distinct foci found within eukaryotic cells that are involved in mRNA regulation and turnover within the CYTOPLASM

274
Q

What vein is grafted for coronary artery use and where is it taken from?

A

Great saphenous vein is taken just inferiorlateral to the pubic tubercle

(keep in mind the left internal mammary (thoracic) vein is preferred over the saphenous but often times is already used or not available)

Right near the femoral triangle

275
Q

Name of gastric bypass and what is a common occurance afterwards due to deficiency of what?

A

Roux-en-Y Gastric Bypass

Can result in Small Intestinal Bacterial Overgrowth (SIBO)

Enteric bacteria produce Vitamin K and folate, which inhibit proliferation of surrounding pathogenic bacteria and digest unabsorbed dietary sugars

See deficiency in A,D,E,B12,Iron but increase in vit K, folate

276
Q

Clinical manifestations of Factor V Leiden Mutation

A

DVT, central vein thrombosis, Cerebral vein thrombosis and recurrent pregnency loss

-Remember that Factor V Leiden produces a mutant factor V that is resistent to inhibition by protein C.

This is the most common cause of hypercoagulability in caucasions

277
Q

B12 deficiency affects what part of the spinal cord?

A
  1. Dorsal column of the spinal cord
  2. Lateral corticospinal tracts
  3. Axonal degeneration of peripheral nerves

Called subacute combined degeneration. This is due to abnormal myelin synthesis

278
Q

Dimpling in breast cancer is caused by what?

A

Involvement of the suspensory ligaments of the breast (also called Cooper Ligaments)

279
Q

Entacapone use and mechanism

A

COMPT inhibitor that increases bioavailability of levodopa by inhibiting its peripheral methylation. If you combine levodopa and Entacapone, you can prolong its duration in the periphery

remember levodopa is the precursor of Dopamine and can cross the BBB

280
Q

How does shigella infect the body?

A

Shigella cannot bind all intestinal cells. Instead, shigella can bind microfold (M) cells at the base of the mucosal villi within a Peyers Patch region of the ileal mucosa.

Shigella endocytoses through the M cell and lyses the endosome, multiplies and then spreads laterally inside other epithelial cells

Infectious dose is VERY LOW and it is resisant to gastric acids

Causes bloody diarrhea

281
Q

Retinal artery occlusion. What is the pathway of the thrombus to get lodged there?

A

The most common cause is from an atherosclerotic plaque in the internal carotid. This then travels to the opthalmic artery and then to the retinal artery.

Common cause of acute, painless, monocular vision loss

282
Q

t(14:18)

A

follicular lymphoma

overexpression of BCL2 (chr 18) to IgG heavy chain (chr14)

BCL2 keeps Cytochrome C from exiting the nucleus and therefore makes the cell immortal by evading apoptosis

283
Q

Mechanism behind Huntington Disease

A

Trinucleotide repeat of CAG on huntington gene that encodes for huntingtin protein. Expansion of the polyglutamine region results in a gain of function mutation.

Autosomal Dominant

Transcriptional repression is one of the mechanisms by which the huntingin protein cuases disease. See increased histone deacetylation, silencing the genes necessary for neuronal survival

284
Q

Use of ACE-I/ARB during PREGNANCY

A

ACE/ARB result in low Angiotensin II that results in fetal renal maldevelopment which leads to decreased diuresis and OLIGOHYDRAMNIOS

If oligohydramnios is severe, can result in pulmonary hypoplasia and skeletal defects (POTTER SEQUENCE)

Low ANG II can also result in hypocalvaria (hypoplasia of skull bones)

285
Q

20 year hx of alcohol abuse with abd pain, diarrea, memory problems, dermatitis..what is dx…what are common findings and what is deficient and what reactions is it involved in?

A

Pt has Pellagra…a deficiency of niacin (B3)

Pellagra: Diarrhea, Dementia, Dermatitis (3 D’s of B3)

Niacin is a precursor for NAD and NADP, two important cofactors for many dehydrogenase and reductase enzymes. Often found deficient in ALCOHOLICS

Cofactor for:

1. Isocitrate dehydrogenase

2. Malate dehydrogenase

3. alpha-ketoglutarate dehydrogenase complex

4. pyruvate dehydrogenase

286
Q

BP 175/68. whats the deal?

A

This is elevated systolic blood pressure with normal diastolic blood pressure. Called isolated systolic hypertension and is often seen in pts over 60 and is responsible for 60-80% of hypertension in this age group.

Increased arterial stiffness is a common manifestation of aging including increased aortic stiffening

287
Q

Volume of distribution

A

Vd=amount of drug given(mg) / plasma concentration of drug (mg/L)

This is a hypothetical amount of fluid that the drug would need to be uniformly distributed into to produce an observed plasma concentration

1. Low –> blood –> large charged molecules, plasma protein bound (3-5L)

2. medium –> ecf –> small hydrophilic molecules (14-16L)

3. high –> all tissues including fat –> small lipophilic molecules, esp if bound to tissue protein (41L)

288
Q

Pseudamonas pneumonia is common in what pts and what is the tx? and what is NOT the tx

A

Common in burn pts and the treatment is:

ticardillin, pipercillin, cefapime, ceftazadime, cipro, levo, carbapenems

NOT moxifloxacin, NOT doxycycline, NOT ceftriaxone, NOT TMP-SMX

289
Q

embryologic derivative of the spleen

A

mesoderm

also muscles, CT, cartilage, CV system, lymphatic system, blood, internal genitalia, kidneys, ureters

MOST gut tissue is endodermal, but the spleen is unique as it is derived from mesenchymal tissue

290
Q

General functions of the hypothalamic nuclei

ventromedial

lateral

anterior

posterior

arcuate

paraventricular

supraoptic

suprachiasmatic

A
291
Q

decreased sensation over 5th finger digit and flattened hypothenar eminence (triceps reflex normal) lesion is where?

A

Hook of the Hamate

This is an ulnar neuropathy. The nerve enters the wrist between the hook of the hamate and the pisiform bone through guyon’s canal

MOST often the ulnar nerve is injured at the elbow “funny bone”

292
Q

Process by which DNA is damaged and repaired after UV light exposure

A

UV exposure creates thimadine dimers –> endonuclease complex recognizes deformed helix –> single strang cleavage on both sides of damage –> damaged DNA is discarded –> DNA polymerase synthesizes replacement –> DNA ligase seals gap

293
Q

Choosing types of insulin (fast/slow/basal/prandial)

A
294
Q

Pain at the front of the neck initilly attributed to illness 1 week ago. Thyroid gland VERY tender with decreased idiodine uptake. ESR 105.

What is it and what is the pathological change?

A

Subacute granulomatous thyroiditis (de Quervain thyroiditis) typically follows a viral illness. Pts may experience transient hyperthyroid phase that is characterized by elevated T4/T3 suppressed TSH and decreaed iodine. Resolves within 6 months

Pathology: early neutrophilic infiltrate with microabscess formation. This is replaced with lymphocytic infiltrate with macrophages and giant cells

295
Q

ADH is synthesized where?

A

paraventricular and supraoptic nuclei of the hypothalamus

296
Q

Treatment of pinworms (enterobiasis)

A

-Bendazoles (worms are bendy) (Albendazole)

These are the most common helminth infection in children ages 5-10 in the usa.

Dx: scotch tape test looking for eggs

297
Q

Diverticulosis vs Diverticulitis

A

Diverticulosis: is an outpouching (false diverticulum) caused by increase pressure (pulsion) during bowel movements. This pressure herniates through areas of focal weakness in the muscularis (false diverticula). Most commonly affects the sigmoid colon and can present with painless hematochezia

Diverticulitis: when diverticula become inflammed causing LLQ pain, low grade fever, constipation or diarrhea

298
Q

D-xylose test

A

passively absorbed in small intestine. Blood and urine levels decrease with mucosal defect or bacterial overgrowth.

it is a monosaccharide that is abosrbed directly without the action of pancreatic enzymes and can be used to test for brush border absorptive function independent of pancreatic function

299
Q

What is a mediator that would increase ESR in a pt?

A

IL-1

IL-6

TNF-alpha

These three cytokines (which are released by neutrophils) mediate systemic inflammatory response and stimulate hepatic secretion of acute phase proteins (fibrinogen). When fibrinogen specifically is activated, it causes RBC to form stacks roleaux that sedimate faster than normal RBC (thus the increased ESR)

300
Q

This kiddo presents with recurrent skin rashes that come about after eating certain foods. What is it and what is causing it?

A

This patient has atopic dermatitis (eczema) that can be triggered by various environmental stimuli. Intense pruritis is the hallmark of this disorder and dx cannot be made without it.

Of note, the allergic triad of Asthma, Atopic Dermatitis, and Allergic Rhinitis is very common, esp in kids with a family history

301
Q

life of a B-cell and what is happening specifically in this photo?

A
  1. Precursors proliferate and mature in the BONE MARROW
  2. Mature B-cells migrate to lymph nodes, where they are exposed to antigens (some B-cells become activated to plasma cells expressing IgM through T-cell independent process)
  3. B-cells migrate to lymphoid follicles in the lymphoid cortex where they from germinal centers
  4. Isotype switching then occurs in the germinal centers (this requires CD40 binding to CD40L expressed by activated T-cells
302
Q

Fetal-Placental aromatase deficiency manifests how?

A

Both maternal and fetus (if female) virulization

Females will have normal internal genitalia but male external genitalia due to exposure to testosterone (cliteromegaly)

303
Q

IgE independent vs IgE dependent mast cell degranulation

A

IgE independent: opioids, radiocontrast dyes, vancomycin can induce mast cell degranulation by PKA or P13 kinase

IgE dependent: this is more common with environmental exposures such as foods or vespid stings or with meds such as beta-lactams and sulfonamide antibiotics

304
Q

Diabetic mononeuropathy mechanism

A

caused by ischemic nerve damage to CN3 that presents with acute onset diplopia. The affected eye assumes a “down and out” position due to unopposed pull by superior oblique and lateral rectus.

The pupil is NORMAL and REACTIVE

305
Q

What is it and what is the treatment of this?

Specifically, what treatment activates a nuclear transcription factor?

A

This is a classic example of psoriasis that is made up of sharply demarcated salmon plaques covered with a losely adherent silvery scale that commonly affects the extensor surfaces.

Tx: topical Vitamin D (in form of calcipotriene or calcitriol). This has been shown to inhibit T-cell proliferation. This binds and inhibits keratinocyte proliferation and stimulates keratinocyte differentiation

306
Q

Long term vs short term treatment of Rheumatoid Arthritis

A

Short term: NSAIDS, Glucocorticoids

Long term: Disease modifying anti-rheumatic drugs (DMARDs) Methotrexate, sulfasalazine, hydroxychloroquinine, minocycline, TNF-a inhibitors

307
Q

Enoxaparin

A

Low molecular weight heparin that is used as first line treatment in a pt with a DVT

This also does NOT cross the placenta and are a bridge to Warfarin tx that will be the longstanding tx of DVT

308
Q

hypoketoic hypoglycemia

A

consistent with a defect in the fatty acid b-odixation in the mitochondria

The most common defect is medium chain acetyl-CoA dehydrogenase deficiency (MCAD)

Most pts are asymptomatic until they experience a significant fasting period

309
Q

Elevated acetylcholinesterase in amniocentesis?

A

Most consistent with a neural tube defect.

See an increase in alpha-fetal protein as well as ACh

NTD occur because of failure of fusion of the neural plate during the 4th week of development

AFP can cross the placenta and is detectable in the maternal serum. ACE cannot

310
Q
A
311
Q

6 causes of Gout (4/2)

A

Increased production

  1. Tumor Lysis Syndrome
  2. Myeloproliferative Disease
  3. Primary gout (idiopathic)
  4. Hypoxanthine Guanine phosphoribosyl transferase deficiency

__________

Decreased clearance

  1. Chronic kidney disease
  2. Thiazide/Loop Diuretics
312
Q

What is this called? And what does it come from?

pt has easy fatigability. hx of gastrectomy due to nonhealing gastric ulcer, unsteady gate

A

Subacute combined degeneration (B12 deficiency)

-Can occur from pernicious anemia, gastectomy, strict vegan diet

pts also develop atrophic glossitis

313
Q
A
314
Q

What are the three stages of Pertussus infection

A
  1. Catarrhal Phase: similar to many URT infections
  2. Paroxysmal Phase: severe coughing spells with classic whoop
  3. Convalescent phase: cough improves

This should not be a problem with immunizations, but if not immunized, becoming more common. This can happen in adults as well if they decide not to be immunized or get boosters

Pertussis: Gram - coccobacillus Bordatella Pertussis

315
Q

What does this lesion represent?

(mass resected from liver)

A

This is a cavernous hemangioma and is the most common benign liver tumor that presents in adults 30-50 years old.

-Cavernous blood filled vascular spaces of variable sizes lined by a single epithelial layer

BIOPSY CONTRAINDICATED because of risk of HEMORRHAGE

316
Q
A
317
Q

S3 and S4 heart sounds

A

S3: this is a ventricular gallop sound heard after S2. Heard during rapid filling of ventricles in diastole. Turbulent blood flow to the ventricles due to increased volume. Seen in pregnancy, heart failure, restrictive cardiomyopathy

S4: this is a atrial gallop sound heard before S1 heard immediately after atrial contraction as blood is forced into a STIFF ventricle. Seen in healthy older adults, ventricular hypertrophy, and acute myocardial infarction

318
Q

What is a good laboratory measure for osteoblast activity?

What is a good lab measurment for osteoclast activity?

A

Osteoblast:

Bone specific Alkaline Phosphatase

Osteoblasts release alk phos as they build bone

Bone specific ALP can be differentiated by that released by the liver by exposing it to heat and easily denaturing it (Boil=Bone)

Osteoclast:

tartrate-resistant alkaline phosphatase (TRAP) (not really used because of cross-reactivity with acid phosphatase from other tissues)

319
Q

Nephrotic syndrome pt has sudden onset abd pain, flank pain, hematuria, left sided varicocele suggests what? and due to what lost in urine?

A

This suggests RENAL VEIN THROMBOSIS due to loss of Antithrombin III in the urine because of the nephrotic syndrome

320
Q

Smooth endoplasmic reticulum secretes what?

A

steroid and phospholipid biosynthesis

321
Q

Resistors in parallel

A

Total Peripheral Resistence= 1/Rtotal=1/R1+1/R2+1/Rn….

Systemic circulation is arranged in parallel

322
Q

Virulence factors of Neisseria Meningitidis and which one specifically is respnsible for the toxic shock and death from the meningitis?

A

1. Lipooligosaccharide (LOS) –> this endotoxin causes sepsis by inducing a systemic inflammatory response caused by an increase in TNF-alpha, IL-1beta, IL-6, and IL-8. Can eventually lead to Waterhouse Friderechson (hemorrhage of adrenals)

2. Polysaccharide Capsule

3. IgA protease

323
Q

spontaneous rest and nighttime angina with associated transient ST elevation has what condition? and can be induced by what drug?

A

This pt has prinzmetal angina (coronary vasospasm)

Can be caused by cocaine, dyhydroergotamine (an ergot alkaloid that is used in migrane headaches because it causes alpha agonist and serotonic agonist), cigarette smoking, amphetamines, triptans

324
Q

Rifaximin use and mechanism

A

a nonabsorbable antibiotic that alters GI flora to decrease intestinal production and absorption of ammonia. Typically used in addition to lactulose (traps NH4+ for excretion) to treat hepatic encephalopathy

325
Q

Blunt aortic injury (from car crash or skiing crash hitting immovable object) causes rupture of aorta where??

A

The aortic isthmus (just past the arch of the aorta) because it is tethered by the ligamentum arteriosum and is relatively fixed and immobile compared to adjacent descending aorta

326
Q

bradycardia med for cardiac pt and what is a common side effect/contraindication

A

The med is Atropine (reversible antagonist of muscarinic ACh receptors) (anticholinergic drug)

A common side effect is eye pain due to flair-up of glaucoma (due to increased intraoccular pressure)

327
Q

Power and how to increase it?

A

Power =1-Beta where beta is False negative (or a type II error)

-Increase power by:

1. increase sample size

2. increase expected effect size

3. increase precision of measurement

328
Q

What is the most common makeup of a complete mole? what is the second most common make up?

A

#1: 46XX (this is one sperm that was 23X that fertilized with inactive egg and replicated itself)

#2 46 XY (this would mean two sperm fertilized an ovum with inactive maternal chromosomes)

Women through the door first

See no fetal structures

increased risk of cancer and choriocarcinoma (1-3%)

329
Q

Where does the trigeminal nerve exit from on the brainstem?

A

The lateral aspect of the midpons at the level of the middle cerebellar peduncle

330
Q

Elementary School Teacher P1G0 has miscarrage at 18 weeks. She reports pain in both knees and feet. What did she have?

A

This is a classic example of parvovirus (a nonenveloped single stranded DNA virus) that is part of the O in TORCHES.

Parvovirus responsible for erythema infectiosum = 5th disease (five fingers when slapped)

Can cause hydrops if exposed in early pregnancy (due to interruption of erythropoeisis leading to profoudn anemia and CHF)

331
Q

What does this guy have and what is the inheritance of it? And what are 5 ways it presents?

A

This is neurofibromatosis type 1 and it is Autosomal Dominant

-This is a mutation on the NF-1 gene on Chr 17

Presentation:

1. Cafe-au-lait spots

2. Neurofibromas

3. Lisch Nodules (hammartomas of iris)

4. Congenital pseudoarthritis

5. Other associated tumors (such as gliomas, astrocytomas, pheochromocytomas)

332
Q

Atrial Fibrillation ECG, pathophys, who is creating the ventricular contraction rate?

A

Absent P-waves. Iregularly irregular R-R intervals, narrow QRS

-Abberent electrical impulses that arise within regions of heightened atrial excitability. This leads to electrical remodeling of the atria with the development of shortened refractory periods and increased conductivity

*The AV node refractory period regulates the number of atrial impulses that reach the ventricle*

333
Q

which two AA are exclusively ketogenic and will not be converted into lactic acid? What is a good example of a condition that would need only these two AA?

A

Leucine, Lycine

These would need to be increased in a patient with a pyruvate dehyrogenase deficiency (cannot enter the TCA cycle), thus these pts need to be exclusively in Ketogenesis. This disorder causes lactic acidosis and neurologic defects

334
Q

In a subarachnoid hemorrhage what drug can be used to prevent cerebral vasospasm? and why do you want to prevent this?

A

Nimodipine: a calcium channel blocker can be given after a SAH (that is often a result of a ruptured berry aneurysm in ADPKD)

-Vasospasm occurs in SAH in 20-30% of pts and leads to AMS, focal neurological deficits. This most often occurs 7-8 days following a SAH

335
Q

Phencyclidine intoxication (PCP)

A

Moderate amounts cause dissociative symptoms. Causes hallucinations and violent behavior with higher doses

NMDA antagonist and secondarily it inhibits the reuptake of NE, 5HT, DA and also has effects on sigma-opioid receptors

Think PCP BAD drug = antagonist

336
Q

Pathophys and labs of primary hyperaldosteronism

A

Characterized by increased sodium reabsorption from the collecting tubule. This creates a negative charge in the lumen pulling H+ and K+ into the lumen. This result in a hypernatremic, hypokalemic, metabolic alkalosis .

The increased H+ in the lumen results in an increase HCO3-/Cl exchanger which promotes bicarb production

The hypernatremia and pedal edema are rarely seen due to aldosterone escape. This is due to increased plama volume leading to increased renal blood flow leading to naturesis which limits the net sodium retention

337
Q

A reliable test is ___?

A

reproducible (gives very similar results whether or not they are accurate)

=precise

338
Q

polyethylene glycol mechamism and use and what is a similar mechanism in the gut?

A

provides osmotic load to draw water into the lumen to treat constipation

Similar mechanism to lactase deficiency which creates osmotic diarrhea

339
Q

Fructose 2-6 Biphosphetate does what and what would be the impact if it was blocked?

A

Helps to regulate PFK-1 (glycolysis) and fructose 1-6 biphosphetase (gluconeogenesis)

Insulin activates PFK2, which then increases PFK1 via fructose 2-6 biphosphetate

Glucagon does the opposite

high concentrations of fructose 2-6 biphosphate inhibit gluconeogenesis leading to decreased conversion of alanine and other gluconeogenic substrates

340
Q

pt with erosion of dental enamel and elevated serum amylase, and calluses on the dorsum of their hands has what condition?

A

Bulemia Nervosa

Tx: SSRI, nutritional rehab, CBT

341
Q

hydroxyurea use and mechanism

A

used in sickle cell anemia (esp in frequent pain crises)

mechaism: increases fetal hemoglobin (HbF). This confers protection against the polymerization of sickle cells.

342
Q

Diastolic heart failure due to restrictive cardiomyopathy with associated ventricular hypertrophy is indicative of what?

A

Cardiac amyloidosis due to deposition of AL deposits (Ig light chain)

blue arrow is normal myocardial cells

black arrow is an area of the myocardium infiltrated with amorphous and acellular pink material

343
Q

walk through the lifecycle of hepatitis B

A

It is part of the DNA containing Hepadnaviridae and consists of a spherical double layered particle that has a nucleocapsid pore covered by an outer lipoprotein envelope (containing surface HBsAg)

*A partially circular DNA molecule that replicates through reverse transcriptase*

Unique in the fact that it is a DNA virus that replicates via reverse transcription

344
Q

Most common location for epistaxis

A

The vascular watershed area of the nasal septum known as Kisselbach plexus

This region has several significant anastamoses:

  1. septal branch of anterior ethmoidal artery
  2. lateral nasal branch of sphenopalantine
  3. septal branch of superior labial artery (branch of the facial artery
345
Q

2 week old with persistent painless swelling of R scrotum with normal testicle descent

A

This is a hydrocele. Specifically, a congenital-communicating hydrocele, a collection of peritoneal fluid within the tunica vaginalis.

This happens because the processus vaginalis remains patent and allows peritoneal fluid to accumulate in the tunica vaginalis

Very common in newborns and presents as painless scrotal swelling that transluminates

346
Q

Antibiotic that can cause serotonin syndrome?

A

Linezolid: used to treat gram + bacteria, particularily VRE and MRSA

Linezolid has MAOI activity and thus when combined with a pts SSRI, they can present with serotonin syndrome (usually need two meds to get this syndrome)

347
Q

1 year old AA presents with painful swelling of hands and feet

A

This patient has sickle cell anemia and is presenting with a vasoclussive episode of dactylitis (hand-foot syndrome)

the repeated sickling of RBC leads to an intra and extra-vascular hemolysis resulting in an increased billirubin and lactate dehydrogenase as well as a DECREASED HAPTOGLOBIN (haptoglobin binds circulating hemoglobin and reduces excretion of free hemoglobin)

This is a common presentation of SCD in young patients

348
Q

What is the cause of gallstones during pregnency?

A

Pregnancy and the use of oral contraceptives predispose women to gallstone formation with 5-10% of all pregnancies developing cholelithiasis

Estrogen –> upregulate HMG-CoA reductase activity, which causes bile to be supersaturated with cholesterol

Progesterone –> reduces bile acid secretion and slows gallbladder emptying

*when the gallbladder is hypomobile or there is more cholesterol in the bile, there is a prediliction of the bile to precipitate

349
Q

Schizoaffective Disorder

A

Dilusions or hallucinations for >2 weeks in the absense of a mood episode (manic or depressive)

(vs schizophrenia that is lasting >6 months and lacks significant mood symptoms)

350
Q

Aspiration pneumonia sitting vs laying flat

A

Sitting –> base of R lower lobe (can be left too but less likely

Laying flat –> posterior segment of the right upper lobe (and superior segment of lower lobe)

351
Q

Latanaprost use

A

This is a glaucoma med that is a prostaglandin analog that increases outflow of aqueous humor via uveoscleral outflow

Remember that open angle glaucoma is caused by increased production of aqueous humor and decreased secretion through canal of Schlemm and Uveoscleral outflow tract. It is characterized by increased intraoccular pressure. The symptoms develop over decades and involve loss of peripheral vision

352
Q

label these parts of the eye

A
353
Q

side effects of statins and what needs to be checked before they are started?

A

Side effects are muscle aches and hepatotoxicity. Need to check LFTs before starting a statin. Increased LFTs (more than three times the upper limit) only occurs in 1% of pts put on statins but can be eliminated with cessation of the drug

354
Q

stomach ulcer vs erosion

A

Gastric erosions: are mucosal defects that do NOT penetrate through the muscularis mucosa. These tend to occur in acute errosive gastropathy that can be caused by NSAIDS, curling, cushing, smoking, ETOH

Gastric ulcers: penetrate through the muscularis mucosa into the submucosa. Causes are NSAIDS (inhibiting prostaglandin synthesis), H.Pylori

355
Q

Gallstone Ileus

A

fistula between gallbladder and GI tract –> air in biliary tree –> passage of gallstones into GI tract –> obstruction of iliocecal valve by stone

356
Q

label this photo

A

Contraction results in shortening of H and I bands and between Z lines (HIZ shrinkage)

The A band remains the same length (A band is Always the same length)

357
Q

metabolic effects of human placental lactogen

A

responsible for gestational diabetes

358
Q

common pathological causes of gynecomastia

A
359
Q

Name 4 different drugs used in Parkinson’s Disease and where they act at

A
  1. Entacapone, Tolcapone (COMT inhibitor, keeps levodopa in periphery)
  2. Carbidopa (prevents DOPA carboxylase from breaking down levadopa)
  3. Bromocriptine, Pramipexole (Dopamine agonist in CNS)
  4. Tolcapone in CNS (COMT inhibitor that crosses BBB, keeps Levodopa from being broken down)
360
Q

Ivabradine mechanism (cardiac drug)

A

lengthens the slow diastolic depolarization phase by inhibiting the Ifunny channels (mixed Na/K inward current)

Accounts for automaticity of SA and AV nodes. The slope of phase 4 in the SA node determines HR

361
Q

Cardiac abnormalities associated with:

Down

DiGeorge

Fredreich

Kartaganer

Marfan

Tuberous Sclerosis

Turner

A
362
Q

Blood supply to cerebral hemispheres

A
363
Q

Important landmarks of the CXR

A
364
Q

Mechanism of common antiseizure meds:

phenytoin/carbamezapime

Valproic Acid

Benzoz/Phenobarb

Levetiracetam

Ethosuximide

A
365
Q

What is this picture showing and what are some other symptoms that are associated with this disease

A

This is a Kayser-Fleischer ring and is associated with Wilson’s disease. The rings are caused by the deposition of copper within the basement membrane of the cornea

Wilson’s disease is a rare autosomal resessive disease that affects individuals 5-40 years old. The mutation on chr 13 (ATP7B gene) hinders copper metabolism. Eventually the copper leaks from the injured hepatocytes and enters the circulation where it can deposit in the cornea and the basal ganglia

tx: D-penicillamine

366
Q

Tourette Disorder

A

multiple motor and >1 vocal tic (not necessarily concurrent) that lasts > 1 year

The vocal tics can be coughing, grunting, throat clearing, to blurting out inappropriate comments (10-20% of pts)

typically presents in boys 6-15

comorbid ADHD and OCD is common

367
Q

Anticipation

A

Pts who receive an abnormal gene from their father tend to develop the disease at a younger age. Common in Huntington, Fredrich Ataxia, Mytonic dystrophy, and Fragile X syndrome

CAG repeats increase during spermatogenesis in the HTT gene on chr 4

The higher number of CAG repeats corresponds to more severe sx

HD is transmitted in AD fashion with COMPLETE PENETRANCE

368
Q

HA, fever, mental staus change in 9 year old girl with seizures. Speech inappropriate choice of words and difficult to understand. Pt most likely has what?

A

HSV-1 encephalitis: infection of the oropharynx travels through the olfactory tract or trigeminal nerves to invade the temporal lobes

Pt gets primary oropharynx infection that travels retrograde down the trigeminal nerve. It lives in the trigeminal ganglion. Upon reactivation, it can travel back down anterograde to the face or can continue travel up the nerve into the brain causing encephalitis

369
Q

Permissiveness

A

Presence of substance A is required for the full effects of substance B

ex: cortisol on catecholamine responsiveness

370
Q

accidental exposure to chemicals, pt presents with dizziness, SOB, palpitations, flushed skin, tachnypnic. Pt is given amyl nitrate to inhale. What poisoning does this person have and what is the mechanism behind amyl nitrate

A

This pt has cyanide poisoning that presents with reddish skin discoloration, tachnypnea, HA, tachycardia, N/V, confusion, weakness

Lab would show lactic acidosis with narrowing of venous PO2 gradient (resulting from inability of tissue to extract O2)

Tx: Inhaled Amyl Nitrate –> oxidises Fe2+ to Fe3+ generating methemeglobin. This is incapable of binding oxygen but can bind cyanide, which will free it from cytochrome oxidase and limit its toxic effects

(can also give thiosulfate and hydroxycoabalin (a precursor for B12))

371
Q

Side effects of major diuretic classes:

Loops

Thiazides

Potassium Sparing

Carbonic Anhydrase

Osmotic (mannitol)

A
372
Q

kidney stones in dehydrated pt due to excess fluid loss from ileostomy (not via urine). What type of stones are these and what is the mechanism behind them?

A

This is a uric acid stone that often appear rhomboid in shape. They are radiolucent. Risk factors include low pH and low urine volume.

Pts with chronic diarrhea have increased loss of bicarb and are in a state of chronic metabolic acidosis. The kidneys compensate by increasing H secretion into tubule and increasing bicarb reabsorption in collecting ducts. This makes urine acidic and increases conversion of soluble urate salts into insoluble uric acid

373
Q

Biliary Atresia

A

partial, complete, or progressive obstruction of extrahepatic bile ducts.

Usually infants appear healthy at birth and then undergo progressive jaundice, dark urine, acholic (pale) stools due to excessive renal excretion of bilirubin and lack of bile

Labs: elevated direct bilirubin (direct =conjugated)

374
Q

Transesophageal echocardiography (TEE). If at mid esophagus level, what is anterior and what is posterior to the probe

A

Anterior: Left Atrium

Posterior: Descending Aorta

375
Q

Lymphatic drainage of rectum above and below dentate line

A

Above dentate –> internal iliac/ inferior mesenteric

Below dentate –> inguinal

As a general rule of thumb, the lymph drainage follows the blood supply in this area

376
Q

RBC without central pallor. How present? What disease? and have what additional findings?

A

Hereditary Spherocytosis: often present with increased indirect billirubin, anemia, reticulocytosis.

The blood smear will show large round cells without any central pallor. This is an AD disease that causes defective membrane binding to due ankryin, spectrin, band 3 mutations

**The most important lab finding is an increase in mean corpuscular hemoglobin concentration MCHC**

377
Q

Long time smoker presents with lung mass and sodium of 120 and plasma osmo 250 (low). What is initial problem and what is it causing?

A

The pt has small cell carcinoma of the lung from his smoking habit. This cancer often presents with paraneoplastic syndrome where there is excess ADH produced –> SIADH

This leads to increased water reabosrption by the kidney leading to a temporary subclinical hypervolemia. This shuts down RAAS and stimulates natruetic peptides which then dump Na in urine (naturesis). As a result SIADH pts have normal volume with low sodium = euvolemic hyponatremia

Volume overload sx are NOT seen

378
Q

cyproheptadine

A

used to treat serotonin syndrome (5HT-2 receptor antagonist)

379
Q

med that can treat both BPH and HTN

A

Doxazosin (alpha-1 blocker) Relaxes smooth muscle leading to decreased peripheral resistance and lowering BP. Also relaxes smooth muscle of bladder neck and prostate leading to a decrease in urinary obstruction caused by BPH

380
Q

sphingomyelinase deficiency. what disease and what is progression of disease?

A

Neimann-Pick Disease is an AR disease caused by deficiency in sphingomyelinase causing excess spingomyelin. These cells look enlarged and foamy. These lipid-layden foam cells accumulate in the spleen and liver causing hepatosplenomegaly and CNS causing neurodegeration

Death by age 3

Also have cherry red macular spot

381
Q

abrupt onset hematuria in a otherwise healthy pt with a family hx of sickle cell disease

A

Renal papillary necrosis due to underlying sickle cell trait

symptoms usually involve dark or bloody urine with colicky flank pain due to uteral obstruction (with no CVA tenderness)

Can also be seen in pts with DM, analgesic nephropathy, or severe obstructive pyelonephritis

SAAD papa with Papillary necrosis

S: sickle cell disease/trait

A: acute pyelonephritis

A: analgesics (NSAIDS)

D: DM

382
Q

High frequency deep brain stimulation for pts with Parkinsons in an attempt to suppress neuronal activity

A

In Parkinsons there is degeneration of the nigrostriatal pathway. This leads to excessive excitation by the globus pallidus by the subthalamic nucleus, which in turn causes excessive inhibiton of the thalamus. Reduced acvitiy of the thalmus on the cortex results in bradykinesia and ridgitiy

DBS of the globus pallidus internus or subthalamic nuclei inhibits firing of these nuclei, which eventually results in thalamo-cortical disinhibition with improved mobility (think want to take direct leash off of the thalamus and let it fire)

383
Q

Myoclonic epilepsy with ragged red fibers (MERRF)

A

This is a very rare mitochondrial myopathy, where abnormal mitochondria accumulate under the sarcolemma of muscle fibers

This is one of the red ragged fiber diseases

*Remember that sperm mitochondria are NOT pass into the ovum during fertilization and thus all mitochondrial diseases are passed through the maternal lineage

384
Q

label some features of this photo

A
385
Q

what are some common signs of liver cirrhosis

A

jaundice

Portal Hypertension

esophageal varicies

splenomegaly

ascites

caput medusa

anorectal varices

Hyperestrinism

testicular atrophy

spider angioma

gynecomastia

386
Q

somatic vs germline mosaicism

A

somatic –> mutations that arise from mitotic errors AFTER fertilization and propagates through multiple tissues or organs. CANNOT be passed on to next generation. Ex: leukocytes with 40% 46X and 60% XX seen in Turner syndrome

germline –> mutation only in egg or sperm cells. The mutation would be in EVERY cell of the body

387
Q

Kartagener syndrome triad

A
  1. situs invertus
  2. chronic sinusitis

3. bronchiectasis (photo)

Also have primary ciliary diskinesia –> impaired sperm motility (infertile)

388
Q

6 classes of drugs that can cause anticholinergic effects (cant see, cant spit, cant pee, cant shit)

A
  1. TCA (amitriptyline)
  2. Antihistamines (diphenhydramine, doxylamine)
  3. 1st gen antipsychotics (chlorpromazine)
  4. 2nd gen antipsychotics (clozapine)
  5. Antiparkinsons drugs (benztropine, trihexyphenidyl)
  6. Belladonna alkaloids (atropine)
389
Q

A single injection of hCG for an anovulatory PCOS pt has what effect?

A

B-hcG has a very similar structure to LH, so it can stimulate the LH surge by inducing ovulation. This is done when the follicle appears mature but will not be released due to the PCOS

390
Q

Baclofen

A

an agonist at the GABA-B receptor is effective as a monotherapy for the treatment of spasticity secondary to brain and spinal cord disease, esp MS

-can be used for muscle spasms (low back pain)

391
Q

number one risk factor for suicide

A

history of prior suicide attempt

392
Q

nitrates for CP

A

direct vascular smooth muscle relaxation resulting in:

  1. vasidilation of peripheral veins and arteries, predominently vasodilation
  2. decreased LV wall stress due to decreased preload (decreased LV end-diastolic volume and pressure)

3. modest reduction in afterload due to arterial vasodilation

  1. mild coronary artery dilation and reduction of coronary vasospasm
393
Q

foot held in calcaneovalgus position (dorsiflexed and everted)

A

Often arises from injury to the Tibial nerve at the popliteal fossa can cause weakness of foot plantarflexion. Weakness on foot inversion and toe flexion

*Results in sensory loss over sole of foot too*

394
Q

AML pt with increasing R sided HA behind his right eye with nasal stuffiness. This is a biopsy of the right maxillary sinus mucosa

A

Rhizopus species

Tends to affect the paranasal sinuses

Form broad nonseptae hyphae that branch at wide 90 decree angles

Compare to Aspergillus fumigatas BELOW that branch at 45 degrees (V shaped branching

395
Q

Acute Cholecystitis

A

Gallstone obstruction of the cystic duct in 90% of cases. Ingestion of fatty food then causes contraction of the gallbladder and results in a colicky pain

US is the best diagnosis of this however nuclear medicine hepatobiliary screening can be an alternative means when US is inconclusive (this is a more precise way to dx acute cholecystitis). Tracer is injected IV and taken up by hepatocytes and dumped into bile. Images are taken throughout

IF you are using an US however, cholecystitis is more likely to show gallbladder wall thickenin, pericholecystic fluid and positive sonographic murphy sign

396
Q

Niacin can predispose you to?

A

Gout via decreased renal excretion of uric acid, leading to elevated blood levels and increased risk for acute gouty arthritis

397
Q

effects of prolonged glucocorticoid therapy on

adipose

adrenal

bone

liver

immune

skeletal

skin

and what organ has the most increase in protein synthesis?

A

Liver via stimulation of gluconeogenesis and glycogenesis. Both actions lead to an overall increase in liver protein synthesis

398
Q

Kawasaki Disease findings

A

Vasculitis of medium sized arteries that affects children <5 years old. Occurs most often in children of Asian descent

Need fever >5 days and 4 of 5 sx below

A serious complication is coronary artery aneurysms

Can treat kids with ASA!!!!

399
Q

Tibial compartments and their contents (arteries and nerves)

A

Anterior compartment –> foot extensor muscles, anterior tibial artery, deep peroneal (fibular) nerve

Lateral Compartment –> superficial peroneal nerve

Deep posterior compartment –> tibial nerve, posterior tibial artery and vein, peroneal artery and vein

Superficial posterior compartment –> no vessels, gastroc and soleus

400
Q

sickle cell anemia point mutation

A

valine (hyrophobic) for glutamic acid (hydrophilic)

  • The incorporation of this abnormal beta-globin protein into hemoglobin results in the formation of HbS
  • HbS polymerizes at low oxygen tension causing sickling and hemolysis resulting in vascular occlusion
401
Q

cross-sectional study

A

simultaneously measures exposures and outcomes (also known as a prevelence study)

has a “snapshot” design that is frequently used in surveys

402
Q

Hemiballisum effects what part of the brain?

A

Damage to the subthalamic nucleus can decrease excitation of the globus pallidus internus thereby reducing inhibition of the thalamus and results in hemiballismus (wild, involuntary large amplitude flinging movements involving the proximal limb.

-most commonly occurs in the setting of a lacunar stroke

403
Q

major side effects of amiodarone:

cardiac

pulm

endocrine

GI/hepatic

occular

dermatologic

neurologic

A

Remember amiodarone is 40% iodine by weight

amIODarone can cause a lot of alterations in thyroid function

404
Q

what is the effect of propanolol in hyperthyroidism

A

hyperadrenderic mainfestations in severe thyrotoxicosis reflect a generalized increased sensitivity to catecholamines

propanolol:

  1. reduce the pts HR and general feelings of anxiety
  2. reduces conversion of T4 –> T3 by inhibiting iodothyronine deiodinase
405
Q

All 3 prokaryotic DNA polymerases have….but which one has the opposite….

A

All three have the ability to remove mismached nucleotides by their 3’ –> 5’ endonuclease (proofreading)

ONLY DNA Pol 1 has a 5’–>3’ exonuclease activity

406
Q

Phenylephrine

A

selevtive alpha-1 adrenergic agonist that causes marked arterial vasoconstriction causing an increase in vascular resistance and blood pressure

It also decreases pulse pressure and HR and SV (via a baroreceptor-mediated increase in vagal tone

407
Q

Pathophys of an adult having a patent foramen ovale (PFO)

A

after birth when the umbilical cord clamps and there is decreased pulmonary vascular resistance, the right atrial pressure lowers and the left atrial pressure rises. This pushes the septum primum against the sptum secundum, closing the foramen ovale.

Incomplete fusion occurs in 25% of adults resulting in a PFO

This usually remains closed as pressure on the left is higher than the right. Conditions such as valsalva release can cause a transient increase in R atrial pressure that can cause a right to left shunt. This is particularily concerning in patients with hypercoagulability

408
Q

54 year old male generalized weakness and fatiguability and abd discombort. Pallor abd distension massive splenomegaly. Pancytopenia. NO bone marrow can be aspirated

A

This patient has Hairy Cell Leukemia and the smear will present with lymphocytes with cytoplasmic projections.

This is classically seen in older males and it infiltrates the bone marrow and reticuloendothelial system

Classically causes dry tap on aspiration due to marrow fibrosis. Present with massive splenomegaly

409
Q

Mitochondrial Vacuolization

A

The appearance of vacuoles and phospholipid containing densities within mitochondria generally signifies irreversible injury

Simple mitochondrial swelling is a reversible process

Anything that damages the membrane of the cell or mitochondria results in irreversible damage

410
Q

Role of metalloproteases in wound healing

A

MMPs are secreted by several different cell types including fibroblasts, macrophages, neutrophils, synovial cells, and epithelial cells.

They are essential for breaking down collagen and other proteins in the extracellular matrix. It is important in wound healing as it encourages both myofibroblast accumulation around the wound edges and scar tissue remodeling

When this process happens too much, can see wound contracture

411
Q

Doxorubicin toxic effects on heart and what other things can cause this same effect

A

Dilated Cardiomyopathy

Also caused by ABCCCD

A: Alcohol abuse

B: wet Beriberi

C: Coxsackie B

C: chronic Cocaine use

C: Chagas Disease

D: Doxorubicin

D

412
Q

pulsus paradoxus

A

This is a decrease in the systolic blood pressure (>10) during inspiration (normally falls <10 during inspiration)

Seen in cardiac tamponade, croup, asthma, OSA, pericarditis

The fluid in the pericardium causes limitation of right ventricular expansion that is exacerbated by inspiration when venous return is increased. This leads to a bowing of the interventricular septum toward the left ventricle, decreaing LV end diastolic volume and stroke volume

413
Q

quick way to tell 1st generation antihistamines from second generation antihistamines and what are examples of each?

A

1st gen: end in en/ine or en/ate –> diphenhydramine, dimenhydrinate (dramamine), chlorpheneramine. Remeber these can cross the BBB and cause more significant sedation and cognitive dysfunction. Potentially inappropriate for elderly patients, esp those with preexisting conditions or cognitive impairment

2nd gen: end in ADINE Loratadine, fexofenadine, deslortadine

414
Q

lactase deficiency

A

Leads to fermentation of undigested lactose by the gut and increased production of short-chain fatty acids that acidify the stool (decrease pH). Hydorgen gas is also produced, which increaes breath hydrogen content

Also causes osmotic diarrhea due to high amounts of undigested lacose in the feces.

415
Q

Haemophilus Infleunza B conjugate vaccine (Hib)

A

vaccines containing the polysaccharide alone are ineffective in children under 2 due to their immature humoral immunity. Therefore, the polysaccharide is conjugated to a carrier protein to amplify the patients humoral response against the polysaccharide through T-cell recruitment.

The carrier protein is from the Tetanus Toxoid protein or outer membrane of Neisseria Meningitidis. Immunity is then increased via T-cell depednent stimulation of B lymphocytes and the production of memory B lymphocytes

416
Q

Apocrine vs Holocrine vs Merocrine

A

Apocrine: cells secrete via membrane bound vesicles –> mammary glands

Holocrine: cell lysis releases entire contents of the cytoplasm and cell membrane –> sebaceous glands

Merocrine: cells secrete via exocytosis –> salivary glands, apocrine sweat glands (stupid, I know), eccrine sweat glands

417
Q

organophosphate presentation and what is appropriate antidote

A

presentation is DUMBBELSS

(diarrhea, urination, miosis, bronchospasm, bradycardia, excitation of skeletal muscles, lacrimation, sweating, salivation)

If give Atropine –> reverses muscarinic effects (competitive antagonist) but NOT address nicotinic effects and thus can still lead to muscle paralysis

If give Pralidoxime –> reverses both muscarinic and nictonic (only effective in regenrating AChE if given early)

418
Q

Glanzmann Thromblastenia is defective what and what is this normally a taget of (drug)

A

AR disorder caused by defective or deficient Gp IIb/IIIa on platelet surfaces that typically present in childhood with mucocutaneous bleeding (increased bleeding time)

Normally, this is a target of Abciximab

419
Q

label this figure

A
420
Q

Amino acid sequence within collagen

A

Glycine-X-Y–Glycine-X-Y…

Every third AA is glycine.

X= proline

Y=hydroxyproine or hydroxylysine

421
Q

water deprivation test with desmopressin

Central DI

Nephrogenic DI

A

Central responds to Desmopressin

Partial nephrogenic partially responds

Complete nephrogenic does not respond at all

422
Q

When you have been depressed for >2 years what is this called?

A

Persistent Depressive Disorder (dysthymia)

  • need chronic depressed mood for 2 years
  • no symptom free for >2 months

If it is pure dysthmic syndrome, the criteria for major depressive episode was never met

423
Q

when considering the reactions in glycolysis, what is the fastest reaction and what would be the enzyme involved?

A

Fructose-1-phosphate because it bypasses a few major steps in metabolism allowing for it to be faster

specifically, it bypasses PFK, the rate limiting step of glycolysis

As a result, fructose is metabolized by the liver faster than other monosaccharides and is rapidly cleared from the bloodstream following dietary absorption

424
Q

tingling in fingers and toes after multiple bone fractures and blood loss and a given multiple units of blood (packed RBC) and a resulting low serum Calcium. What is the cause of the low calcium?

A

-Prior to storage, packed RBC are mixed with citrate anticoagulant. Infused citrate can chelate calcium, causing hypocalcemia (pins and needles pt is experiencing)

This is more likely when a pt receives 5-6 liters in a 24 hour period

425
Q

HA, Vomiting, papilledema, dry skin, hepatosplenomegaly in 35 year old women with eccentric diet habits

This patient has what? and what are the three different types?

A

Vitamin A toxicity

Acute: N/V, verigo, blurred vision

Chronic: alopecia, dry skin, hepatic toxicity and enlargement, arthralgias, pseudotumor cerebri

Teratogenic: cleft palate, cardiac enlargement, fetal death if in first trimester (reason why need prove two different types of contraception before isotretinoin therapy)

426
Q

Hirschprung Disease involves what part of body specifically?

A

Need to remember that the nerves (meissner and aurbach plexus) migrate CAUDALLY (towards tail)…so the rectum is ALWAYS involved and the sigmoid colon is involved 75% of the time

This migration happens caudally along the vagal nerve

Newborns with Hirschprung fail to pass meconium within 48 hours of birth

427
Q

solitary mass in the R temporal lobe. What is the visual disturbance

A

This is destruction of meyers loop, an results in Left homonomous superior quadrantanopia “pie in the sky”

428
Q

What produced the lesion seen in the picture?

A

Lysosomal digestion of the tissue.

The brain undergoes liquifactive necrosis

The image shows a secton of the brain with a cystic cavity surrounded by gliosis. Release of lysosomal enzymes from these neurons results in degredation of the tissue in the ischemic region. Phagocytic cells move into the area and and remove the necrotic tissue and leave a cavity and astocytes proliferate and create a glial scar (This whole process of complete digestion of necrotic tissue with formation of a gliosis =liquifactive necrosis)

429
Q

What is going on in this child’s brain (in the posterior fossa)

A

This is an Dandy Walker Malformation –> absence or malformation of the cerebellar vermis (red) and cystic dilation of the 4th ventricle (blue) with posterior fossa enlargement

Pts often present with skull enlargement and developmental delay in infancy. Non-communicating hydrocephalus may occur due to atresia of the foramina leusha and magendie resulting in sx of increased ICP

430
Q

Isoproterenol

A

non-selective b-adrenergic agonist that INCREASES cardiac contractility (by acting on B1) and binds to B2 to decrease vascular resistance

B1=B2

431
Q

glyburide

A

sulfonylurea

(close K channels, cause depolarization, Ca enters, release insulin)

This would increase C-peptide levels (indicating that it was endogenous insulin NOT exogenous)

432
Q

Buprenorphine

A

Partial opiod agonist that has low efficacy for mu-opioid receptors, but it binds with high affinity (potency) and can prevent binding of other medications.

Essentially it acts as an opiod receptor antagonist in the presence of other opioids and can precipitate WITHDRAWL in opioid pts with chronic pain

433
Q

Myotonic Dystrophy (Myotonic Type 1 specifically)

A

AD, CTG trinucleotide repeat expansion in the DMPK gene –> abnormal expression of myotonin protein kinase

shows anticipation

Sx: myotonia, muscle wasting, cataracts, testicular atrophy, frontal balding, arrhythmia

“My Tonia, My Testicles, My Toupee, My Ticker”

434
Q

Capitation (insurance)

A

an arrangement where a payer (employee, individual or government entity) pays a fixed, predetermined fee to cover all the medical services required by a patient.

Capitation is the payment structure underlying HMO provider networks

There is an incentive for the provider and the patient to reduce expeses usually by restricting patients to a limited number of providers

435
Q

Why would you give dexamethasone to a premature infant?

A

Premature infants born at <32 weeks are at significant risk of surfactant deficiency. Corticosteroids such as dexamethasone or betamethasone increase surfactant by accelerating maturation of type II pneumocytes.

-These meds are administered to pts who are at risk of preterm labor to decrease the chance of respiratory distress and mortality.

436
Q

Cardiac catheter waveforms and avg values for each chamber

A

Right Atrium: Mean 1-5

Right Ventricle: Sys: 15-30, Dias: 1-6

Pulmonary Artery: Sys: 15-30 Dias 6-12

Left Atrial/PCWP/LVEDP: Mean 6-12

437
Q

Polyol Pathway (sorbitol breaks down from what to what)

A

Glucose –> sorbitol (via aldose reudctase) –> fructose (sorbitol dehydrogenase)

Sorbitol accumulates in tissues where sorbitol dehydrogenase is low or in long standing hyperglycemia where there is high levels of sorbitol. Sorbitol cannot cross the cell membrane and is thus trapped in cells.

Sorbitol increases osmotic and oxidative stress and contributes to the pathogenesis of retinopathy

Basically, glucose overwelms the pathway causing intracellular sorbitol accumulation

438
Q

Paradoxical embolism, what is it and what is the murmur?

A

Emboli that start in the venous circulation and end up lodged in the arterial circulation via a patent foramen ovale or atrial septal defect (DVT –> Middle Cerebral Atery infarct)

Wide and fixed splitting (no change with respiration) of the second heart sound

This can happen as a transient event such as coughing or during early ventricular systole or defecation

439
Q

Uretheral injury with pelvic fracture most likey affects what part of the ureter?

A

The membranous segment of the urethra (right after the prostate) is relatively unsupported by the adjacent tissues and is one of the weakest parts

*This bulbomembranous junction is often injured*

440
Q

Overdose of methotrexate give what?

A

Folinic acid (leukovorin)

Can reverse MTX toxicity if given early. It does not require DHFR to be converted to THF and is therfore unaffected by MTX

Leukovorin is given during high dose methotrexate to “rescue” bone marrow and GI and other tissue from MTX toxicity

Additionally, it potentiates 5FU (used in colorectal cancer regimens)

441
Q
A